You are on page 1of 133

IAS.

NETWORK

Prelims 2020 Mentorship Program Test 5

( 5th OCTOBER )

Time : 2 Hours​ ​ ​ ​Max Marks : 200

Instructions :

● This Test contains ​100 Questions​, Each question has ​four answers​,
and each question is of ​2 marks​.
● All questions carry ​equal marks​, Your total marks will depend only on
the number of correct responses marked by you. For every incorrect
response ​1/3rd​ of the allotted marks will be deducted.
● In case you feel that there is more than one correct response, choose
which you consider to be the best. In any case, choose only one
answer for each question.
● Answer key and Explanation is provided after the questions for self
assessment.
● Attempt within ​2 Hours​, and questions covers most important topics
only, to maintain a good hit ratio.

© IAS.NETWORK
Q1) Which of the following provisions Q3) Arrange the following north east
of the constitution of India can be Indian states in the order from South
amended by a simple majority of the to North direction based on their
two houses of Parliament ? geographical location

1. Conferment of more jurisdiction on 1.Manipur


the Supreme Court 2.Nagaland
2.Abolition and Creation of legislative 3.Mizoram
councils in states. 4.Arunachal pradesh
3.Elections to the parliament and state
legislature. Select the correct answer using the
codes given below
Select the correct answer by using the
codes given below : a.4-2-1-3
b.3-1-2-4
1.1,2 c.1-2-3-4
2.2,3 d.1-3-2-4
3.1,3
4.1,2,3 Q4) In which of the following systems
of government is bicameralism an
essential feature
Q2) With reference to the procedure
for the amendment of the constitution a.Presidential system
of India, consider the following b.Parliamentary system
statements c.Federal system
d.Unitary system
1.The bill can be introduced either by a
minister or by a private member.
2.The introduction of the bill does not
require prior permission of the
president.
3.The bill can be initiated only by lower
house of the parliament.

Which of the statements given Above


is/are correct ?

a.1,2 c.1,3
b.2,3 d.1,2,3

© IAS.NETWORK
1
Q5) Which of the following are the Q7) Consider the following statements
ingredients of the basic structure of the
constitution of India? 1.The term ‘federalism’ has no where
been used in the constitution of India.
1.Unity and Integrity of the nation 2.The states in India have no right to
2.Limited power of parliament to territorial integrity.
amend the constitution
3.Free and fair elections Which of the given above statements
4.Rule of law is/are correct?
5.Effective access to justice
a.1 only
Select the correct answer using the b.2 only
codes given below c.1,2
d.Neither 1 nor 2
a.1,3,4,5
b.1,2,3,4 Q8) Consider the following statements
c.2,4,5
d.1,2,3,4,5 1.The judge’s strength in uniform in all
the High Courts of India.
2.The Judge’s strength for the High
Q6) The system of legal responsibility Court is fixed by the parliament.
of minister is not a feature of Indian
Parliamentary System because Which of the statements given above
is/are correct ?
a.A person who is not a member of
parliament can also be appointed as a.1 only
minister b.2 only
b.The prime minister may be a c.Both 1 and 2
member of any of the two houses of d.Neither 1 nor 2
the parliament
c.The ministers are not required to
counter sign the official act of the head
of the state.
d.The ministers are collectively
responsible to the parliament.

© IAS.NETWORK
2
Q9) Arrange the following High Courts Q11) “Project 75-I” sometimes seen in
in the correct chronological order in the news recently, is associated with
terms of their formation which of the following?

1.Allahabad High Court a. Fighter aircrafts


2.Madras High Court b. Helicopters
3.Gujarat High Court c. Submarines
4.Kerala High Court d. Armoured Fighting Vehicles

Select the correct answer using the


codes given below

a.3-4-1-2
b.2-1-4-3 Q12) Consider the following
c.2-3-4-1 statements with respect to Strategic
d.2-4-3-1 Partnership (SP) model in Defence
acquisition
Q10) With reference to the
impeachment of the President, 1). It intends to encourage
consider the following statements broader participation of the private
sector in the manufacture of defence
1.The impeachment charges can be platforms in India.
initiated by either House of the 2. Only four segments have been
Parliament identified for acquisition under this
2.The nominated members of either Strategic Partnership (SP) route.
house of parliament can participate in
the impeachment of the president Which of the statement(s) given above
3.The constitution does not define the is/are
meaning of the phrase violation of the correct?
constitution
a. 1 only
Which of the statements given above b. 2 only
is/are correct ? c. Both 1 and 2
d. Neither 1 nor 2
a.1,2
b.2,3
c.1,3
d.1,2,3

© IAS.NETWORK
3
Q13) Consider the following Q15) ) “Naresh Chandra Task force” is
statements with respect to Annual associated with which of the
Financial Statement following?

1. It is not only necessary, but also a. Defence management


compulsory. b. Issues with Start-ups
2. Though both the Houses of c. Regulate Fin-tech sector
Parliament discuss the Budget, only d. None of the above
Lok Sabha votes on it.
3. Vote-On-Account covers only the
expenditure to be incurred.

Which of the statement(s) given above


is/are correct?
Q16) ) Consider the following
a. 2 only statements with respect to VAJRA
b. 1 and 2 only Scheme
c. 1 and 3 only
d. 1, 2 and 3 1. It aims to reverse the brain drain
process in India.
Q14) Consider the following 2. It comes under the Ministry of
statements with respect to International Human Resource Development.
Crops Research Institute for the
Semi-Arid Tropics Which of the statement(s) given above
is/are correct?
1.It is an international non-profit
organization that undertakes scientific a. 1 only
research for development. b. 2 only
2. Its headquarters is located in Manila, c. Both 1 and 2
Philippines. d. Neither 1 nor 2

Which of the statement(s) given above


is/are incorrect?

a. 1 only
b. 2 only
c. Both 1 and 2
d. Neither 1 nor 2

© IAS.NETWORK
4
Q17) Central African Republic (CAR), Q19) INSTEX Channel sometimes
sometimes seen in the news recently is seen in the news recently, is a
bordered by which of the following?
a. Payment channel with Iran to
1. Kenya circumvent U.S sanctions
2. Congo b. Passage that connects South China
3. Sudan Sea with East China Sea
4. Atlantic Ocean c. Route to divert investments in Stock
exchanges based on algorithms
Select the correct answer using the d. World‘s busiest shipping traffic
code given below : channel located in Red sea

a. 2 and 3 only
b. 1, 2 and 3 only
c. 1, 2, 3 and 4 only
d. None

Q18) Consider the following Q20) ) “CAR – T Therapy”, sometimes


statements with respect to Pradhan seen in the news recently, treats
Mantri Kisan Samman Nidhi which of the following diseases?
(PM-KISAN)
a. Tuberculosis
1. Under this scheme, farmers having b. Cancer
cultivable land up to 5 hectares will be c. AIDS
provided income support at the onset d. None of the above
of every harvest year.
2. Funded by the Government of India,
this income support will be transferred
directly into the bank accounts of
beneficiaries.

Which of the statement(s) given above


is/are correct?

a. 1 only
b. 2 only
c. Both 1 and 2
d. Neither 1 nor 2

© IAS.NETWORK
5
Q21) ) Consider the following Q23) ) Consider the following
statements with respect to Railguns statements with respect to Fiscal
sometimes seen in the news Deficit (FD)

1. It uses electricity instead of 1. It is the difference between the


gunpowder to launch projectiles. Revenue Receipts plus Non-debt
2. It eliminates the need to store Capital Receipts (NDCR) and the total
explosives aboard warships. expenditure.
3. United States has recently 2. It is reflective of the total borrowing
developed the world‟s first requirements of Government.
ship-mounted electromagnetic Rail
gun. Which of the statement(s) given above
is/are correct?
Which of the statement(s) given above
is/are correct? a. 1 only
b. 2 only
a. 2 only c. Both 1 and 2
b. 1 and 2 only d. Neither 1 nor 2
c. 2 and 3 only
d. All of the above

Q22) ) Consider the following Q24) ) The Renke Commission and


statements with respect to Pradhan the Idate Commission sometimes
Mantri Shram-Yogi Maandhan seen in the news recently is
associated with which of the
1. It is a mega pension scheme for the following?
unorganised sector workers with yearly
income up to Rs. 50,000. a. De-notified and Nomadic tribes
2. It will provide beneficiaries, an b. 4R approach in Banking
assured monthly pension from the age c. Artificial Intelligence
of 60 years. d. Disaster Response

Which of the statement(s) given above


is/are incorrect?

a. 1 only
b. 2 only
c. Both 1 and 2
d. Neither 1 nor 2

© IAS.NETWORK
6
Q25) Consider the following Q27) Consider the following
statements with respect to Rashtriya statements with respect to Know My
Kamdhenu Aayog India Programme

1. It aims to upscale sustainable 1. It is an initiative for Diaspora


genetic up-gradation of cow resources engagement which familiarizes
and to enhance production and Indianorigin youth with their Indian
productivity of cows. 2. It will look after roots and contemporary India.
effective implementation of laws and 2. It is initiated by the Ministry of
welfare schemes for cows. External affairs.

Which of the statement(s) given above Which of the statement(s) given above
is/are correct? is/are correct?

a. 1 only a. 1 only
b. 2 only b. 2 only
c. Both 1 and 2 c. Both 1 and 2
d. Neither 1 nor 2 d. Neither 1 nor 2

Q26) ) Consider the following Q28) Consider the following


statements with respect to Rashtriya statements with respect to Bharat
Gokul Mission Rang Mahotsav (BRM)

1. It aims to conserve and develop 1. It is an annual International theatre


indigenous breeds of cattle. festival of India.
2. It is run by the Ministry of Agriculture 2. It is being organized by the National
& Farmers Welfare. School of Drama (NSD).

Which of the statement(s) given above Which of the statement(s) given above
is/are correct? is/are correct?

a. 1 only a. 1 only
b. 2 only b. 2 only
c. Both 1 and 2 c. Both 1 and 2
d. Neither 1 nor 2 d. Neither 1 nor 2

© IAS.NETWORK
7
Q29) Consider the following Q31) Consider the following
statements statements with respect to Group of
Monuments at Hampi
1. India has 28 species of turtles, of
which 20 are found in Assam. 1. It is located in the Tungabhadra
2. The black softshell turtle widely basin, Karnataka.
found along Indian coast is listed as 2. Hampi was the last capital of the
critically endangered under the IUCN last great Hindu Kingdom of
Red list. Vijayanagar.
3. It is a UNESCO World Heritage
Which of the statement(s) given above Site.
is/are correct?
Which of the statement(s) given above
a. 1 only is/are incorrect?
b. 2 only
c. Both 1 and 2 a. 2 only
d. Neither 1 nor 2 b. 2 and 3 only
c. 1, 2 and 3
Q30) Consider the following d. None
statements with respect to Government
e-Marketplace (GeM) Q32) Consider the following
statements with respect to Battle of
1. It aims to enhance transparency, Chausa
efficiency and speed in public
procurement. 1. It was fought between Sher shah
2. In its Interim Budget 2019-20, GoI suri and the Mughal Emperor
has decided to restrict this platform to Humayun in 1539.
only public sector enterprises which 2. At the end of this battle, Humayun
have annual net profit of over Rs. left India and went into exile.
50,00 crore.
Which of the statement(s) given above
Which of the statement(s) given above is/are correct?
is/are correct?
a. 1 only
a. 1 only b. 2 only
b. 2 only c. Both 1 and 2
c. Both 1 and 2 d. Neither 1 nor 2
d. Neither 1 nor 2

© IAS.NETWORK
8
Q33) Pobitora wildlife Sanctuary is Q35) Consider the following
located in which of the following statements with respect to Prompt
states? Corrective Action (PCA) framework

a. Assam 1. It is imposed on banks whose


b. Arunachal Pradesh capital, asset quality, profitability
c. Meghalaya and/or leverage do not meet
d. Mizoram pre-specified thresholds.
2. It is applicable only to commercial
Q34) Consider the following banks and not extended to
statements with respect to VISION co-operative banks, non-banking
2030, sometimes seen in the news financial companies (NBFCs) and
recently. FMIs.

1. It expresses government‟s vision for Which of the statement(s) given above


the next two decades, listing thereby is/are correct?
ten most important dimensions in
2030. a. 1 only
2. It was launched by the NITI Aayog. b. 2 only
c. Both 1 and 2
Which of the statement(s) given above d. Neither 1 nor 2
is/are correct?
Q36) “Periodic Labour Force Survey”,
a. 1 only sometimes seen in the news recently,
b. 2 only was prepared by which of the
c. Both 1 and 2 following?
d. Neither 1 nor 2
a. Labour Bureau
b. Society for Labour and
Development
c. NITI Aayog
d. National Sample Survey Office

© IAS.NETWORK
9
Q40) Consider the following
Q37) Senkaku Islands or the Daioyu statements with respect to Samagra
Islands sometimes seen in the news Shiksha Abhiyan
recently, is located in which of the
following? 1. It envisages school education as a
continuum from pre-school to senior
a. South China Sea secondary level and aims to ensure
b. Pacific Ocean inclusive and equitable quality
c. Sea of Japan education at all levels.
d. East China Sea 2. It subsumes three erstwhile
Schemes of Sarva Shiksha Abhiyan
(SSA), Rashtriya Madhyamik Shiksha
Q38) Asian Infrastructure Finance Abhiyan (RMSA) and Teacher
2019 report is released by which of the Education (TE).
following?
Which of the statement(s) given above
a. Asian Development Bank is/are correct?
b. New Development Bank
c. Asian Infrastructure Investment a. 1 only
Bank b. 2 only
d. None of the above c. Both 1 and 2
d. Neither 1 nor 2
Q39) Consider the following pairs
Q41) Consider the following
1. Varahi hydel project – Telangana statements with respect to Rashtriya
2. Kiru Hydroelectric project – Vayoshri Yojana (RVY)
Maharashtra 1. It will provide physical aids and
assisted-living devices for senior
Which of the pair(s) given above is/are citizens belonging to BPL category.
correctly matched? 2. It is being implemented through the
Artificial Limbs Manufacturing
a. 1 only Corporation (ALIMCO).
b. 2 only
c. Both 1 and 2 Which of the statement(s) given above
d. Neither 1 nor 2 is/are correct?

a. 1 only
b. 2 only
c. Both 1 and 2
d. Neither 1 nor 2

© IAS.NETWORK
10
Q42) Consider the following
statements with respect to employment Q44) Consider the following
of women in Indian mines statements with respect to Earthquake
swarm
1. The Mines Act, 1952 prohibited the
employment of women in any part of a 1. It is a series of low magnitude
mine. earthquakes without a discernible
2. Recently, the Ministry of Mines has main shock.
issued a notification, easing the 2. It occurs over a period of time
restrictions and allowed women to ranging from days, weeks to even
work in mines. months.

Which of the statement(s) given above Which of the statement(s) given above
is/are correct? is/are correct?

a. 1 only a. 1 only
b. 2 only b. 2 only
c. Both 1 and 2 c. Both 1 and 2
d. Neither 1 nor 2 d. Neither 1 nor 2

Q43) Consider the following Q45) Global Crisis of Nuclear Waste


statements with respect to Pravasi Report sometimes seen in the news
Kaushal Vikas Yojana recently, was published by which of
the following agencies?
1. It aims to enable a large number of
youths to take up industry-relevant skill a. Atomic Energy Commission
training that will help them in securing b. International Energy Agency
a better livelihood in India. c. International Atomic Energy Agency
2. The scheme has been implemented d. Green Peace
by National Skill Development
Corporation (NSDC).

Which of the statement(s) given above


is/are correct?

a. 1 only
b. 2 only
c. Both 1 and 2 d. Neither 1 nor 2

© IAS.NETWORK
11
Q46) Consider the following Q48) Consider the following
statements with respect to Agri-Market statements with respect to
Infrastructure Fund (AMIF) Indo-Saracenic architecture

1. It is a corpus of Rs. 2,000 crore 1. It is a revival architectural style


under the National Bank for Agriculture used by British architects in India in
and Rural Development (NABARD). the 19th century.
2. It was created for the development 2. The Madras High Court buildings
and up-gradation of agricultural are an example of this style.
marketing infrastructure in rural and
regulated wholesale markets. Which of the statement(s) given above
is/are correct?
Which of the statement(s) given above
is/are correct? a. 1 only
b. 2 only
a. 1 only c. Both 1 and 2
b. 2 only d. Neither 1 nor 2
c. Both 1 and 2
d. Neither 1 nor 2 Q49) Consider the following
statements with respect to
Q47) Consider the following Merchandise Export Incentive Scheme
statements with respect to North (MEIS)
Atlantic Treaty Organization (NATO)
1. Under the Scheme, the government
1. It is an alliance of 29 countries only gives incentives to exporters‟
from North America and Europe. equivalent to a certain percentage of
2. The fundamental role of NATO is to their export value.
safeguard the freedom and security of 2. It is being implemented by the
its member countries by political and Ministry of Commerce and Industry.
military means.
Which of the statement(s) given above
Which of the statement(s) given above is/are correct?
is/are correct?
a. 1 only
a. 1 only b. 2 only
b. 2 only c. Both 1 and 2
c. Both 1 and 2 d. Neither 1 nor 2
d. Neither 1 nor 2

© IAS.NETWORK
12
Q52) Consider the following
Q50) Consider the following statements with respect to Central
statements with respect to National Bureau of Investigation (CBI)
Minorities Development and Finance
Corporation (NMDFC) 1. States cannot deny entry to the CBI
to probe cases referred by the
1. It is a not for profit company Constitutional courts.
established under the companies act 2. Withdrawal of consent to CBI by a
1956. state government can be effected
2. It works under the aegis of Ministry prospectively and not retrospectively.
of Finance.
Which of the statement(s) given above
Which of the statement(s) given above is/are correct?
is/are correct?
a. 1 only
a. 1 only b. 2 only
b. 2 only c. Both 1 and 2
c. Both 1 and 2 d. Neither 1 nor 2
d. Neither 1 nor 2
Q53) Consider the following
statements with respect to e-NAM
Q51) “International Intellectual portal
Property Index” was recently released
by which of the following? 1. It is a pan-Indian electronic trading
portal which seeks to replace the
a. World Bank existing APMC market through a
b. World Economic Forum virtual platform.
c. World Intellectual Property 2. This portal only supports intra-state
Organisation trade and does not support inter-state
d. US Chambers of Commerce transactions.

Which of the statement(s) given above


is/are correct?

a. 1 only
b. 2 only
c. Both 1 and 2
d. Neither 1 nor 2

© IAS.NETWORK
13
Q54) Consider the following Q56) Consider the following
statements with respect to statements with respect to Convention
Intermediate-Range Nuclear Forces on the Conservation of Migratory
(INF) treaty Species (CMS)

1. It is a treaty among UN member 1. It is an environmental treaty under


countries that prohibited development, the aegis of United Nations
testing and possession of conventional Environment Programme (UNEP).
as well as nuclear missiles. 2. India has been a Party to the CMS
2. The treaty did not cover sea based since its inception.
or air launched missiles. 3. 13th Conference of Parties (COP)
3. Recently, U.S.A has decided to of the Convention is going to be
withdraw from the Intermediate-Range hosted by Russia in 2020.
Nuclear Forces (INF) treaty.
Which of the statement(s) given above
Which of the statement(s) given above is/are correct?
is/are correct?
a. 1 only
a. 3 only b. 1 and 2 only
b. 1 and 2 only c. 2 and 3 only
c. 2 and 3 only d. 1, 2 and 3
d. 1, 2 and 3
Q57) Consider the following
Q55) Consider the following statements with respect to Central
statements with respect to Ratoon Asian Flyway (CAF)
cropping
1. It covers a large continental area of
1. It is a form of cultivation in which a Asia between the Pacific and Indian
second crop is allowed to grow from Ocean and the associated island
the remains of one already harvested. chains.
2. Ratooning can be used endlessly as 2. Indian sub-continent is a part of the
the yield and quality increases after Central Asian Flyway (CAF).
each cycle.
Which of the statement(s) given above
Which of the statement(s) given above is/are correct?
is/are correct?
a. 1 only a. 1 only
b. 2 only b. 2 only
c. Both 1 and 2 c. Both 1 and 2
d. Neither 1 nor 2 d. Neither 1 nor 2

© IAS.NETWORK
14
Q58) cVIGIL app is an initiative of Q60) Which of the following
which of the following agencies? birds/animals can be spotted in India?

a. Election Commission of India 1. Amur Falcons


b. Central Vigilance Commission 2. Bar headed Geese
c. Central Board of Indirect Taxes and 3. Black necked cranes
Customs 4. Marine turtles
d. Food Safety and Standards 5. Dugongs
Authority of India 6. Humpback Whales

Select the correct answer using the


code given below
Q59) Consider the following
statements with respect to Solid Fuel a. 1, 3, 4 and 5 only
Ducted Ramjet (SFDR) propulsion b. 1, 2, 3 and 5 only
technology c. 2, 3, 4, 5 and 6 only
d. 1, 2, 3, 4, 5 and 6
1. It is an indigenous technology jointly
developed by the ISRO and DRDO. Q61) Consider the following
2. This technology uses the air as an statements
oxidizer thus eliminates the need for a
propellant. 1. HeliNa is an indigenously
developed air to-land version of Nag
Which of the statement(s) given above missile.
is/are correct? 2. Nag missile is one of the five
missile systems developed by DRDO
a. 1 only under the Integrated Guided Missile
b. 2 only Development Programme (IGMDP).
c. Both 1 and 2
d. Neither 1 nor 2 Which of the statement(s) given above
is/are correct?

a. 1 only
b. 2 only
c. Both 1 and 2
d. Neither 1 nor 2

© IAS.NETWORK
15
Q62) Consider the following
statements with respect to Mera Q64) Consider the following
aspataal initiative statements with respect to
Competition Commission of India
1. It capture patient feedback for the (CCI)
services received at the hospital
through user-friendly multiple 1. It is a statutory body.
channels. 2. It is responsible for enforcing the
2. It is an initiative launched by the Competition Act, 2002 throughout
Ministry of Health and Family Welfare. India.

Which of the statement(s) given above Which of the statement(s) given above
is/are incorrect? is/are incorrect?

a. 1 only a. 1 only
b. 2 only b. 2 only
c. Both 1 and 2 c. Both 1 and 2
d. Neither 1 nor 2 d. Neither 1 nor 2

Q63) Albendazole is a medication used Q65) Which of the following actions


for the treatment of which of the are carried out by Food and
following? Agriculture Organization (FAO)?

a. Seasonal Influenza 1. To help eliminate hunger, food


b. Tuberculosis insecurity and malnutrition
c. Worm infestations 2. Make agriculture, forestry and
d. None of the above fisheries more productive and
sustainable
3. Reduce rural poverty
4. Increase the resilience of
livelihoods to threats and crises

Select the correct answer using the


code given below

a. 1, 2 and 3 only
b. 1, 2 and 4 only
c. 1, 3 and 4 only
d. All of the above

© IAS.NETWORK
16
Q66) Consider the following
statements regarding “Hydro Q68) Consider the following
Seismicity”, statements regarding “Sampoorna
Bima Gram Yojana”
a. It is a phenomenon of occurrence of
multiple tremors under water caused 1. It is implemented by Ministry of
by the slow movement of tectonic Communication.
plates. 2. Its objective is to provide internet
b. It is caused by the emergence of connectivity to villages by establishing
geothermally heated groundwater that optical fibre network
rises from the Earth‘s crust
c. It refers to rise in groundwater Which of the above statement(s) is/are
pressure released in earthquake correct?
swarms due to entering of heavy
rainfall in small fractures in rocks a. 1 only
d. It refers to an induced earthquake b. 2 only
caused by hydraulic fracturing. c. Both 1 & 2
d. Neither 1 nor 2
Q67) Open Acreage Licensing Policy is
related to Q69) Consider the following
statements
a. Exploration of Hydrocarbons
b. Land acquisition under UDAN 1. Algae can be used to generate
c. National Highway Development bio-fuels
d. Swacch Bharat 2. Fungi are natural bio-indicators 3.
Anthrax is caused by bacteria.

Which of the above statement(s) is/are


correct?

a. 1 and 2 only
b. 1 and 3 only
c. 2 and 3 only
d. All of the above

© IAS.NETWORK
17
Q70) Consider the following
statements with respect to Gateway of Q72) Blue Flame Revolution
India sometimes seen in the news recently,
is associated with which of the
1. It is an arch monument built during following?
20th century.
2. It was erected to commemorate the a. Nuclear Energy
Indian soldiers who lost their lives b. Clean cooking fuel
fighting for the British Army during the c. Fishing and its allied sectors
World War I. d. Offshore oil production
3. It is built in Indo-Saracenic style.
Q73) Every Finance Commission has
Which of the statement(s) given above a special theme. What is the special
is/are correct? theme of the Fifteenth Finance
Commission?
a. 1 only
b. 1 and 2 only a. Health and Education
c. 1 and 3 only b. GST Restructuring
d. 1, 2 and 3 c. Poverty and Unemployment
d. Global warming and Climate
Q71) Consider the following Change
statements with respect to World
Sustainable Development Summit
(WSDS)

1. It is the annual flagship event


organised by NITI Aayog.
2. WSDS 2019 will focus on the theme,
'Attaining the 2030 Agenda: Delivering
on our Promise'.

Which of the statement(s) given above


is/are correct?

a. 1 only
b. 2 only
c. Both 1 and 2
d. Neither 1 nor 2

© IAS.NETWORK
18
Q74) Consider the following
statements with respect to e-office Q76) Consider the following
Project statements

1. It is a mission mode project under 1. India is the second largest producer


the National e-Governance Plan. of silk after China.
2. It seeks to replace the physical files 2. India is the largest consumer of silk.
and documents with an efficient
electronic system. Which of the statement(s) given above
3. It is being implemented by the is/are correct?
Department of Administrative Reforms
and Public Grievances (DARPG). a. 1 only
b. 2 only
Which of the statement(s) given above c. Both 1 and 2
is/are correct? d. Neither 1 nor 2

a. 1 only Q77) Consider the following


b. 1 and 3 only statements with respect to Small
c. 2 and 3 only Grants Program (SGP)
d. 1, 2 and 3
1. It provides both financial and
Q75) Dard Aryan tribes sometimes technical support to communities and
seen in the news recently, inhabit Civil Society Organizations to meet
which of the following states? the overall objective of global
environmental benefits secured
a. Jammu and Kashmir through community-based initiatives
b. Odisha and actions.
c. Chattisgarh 2. It is implemented by UNDP on
d. Arunachal Pradesh behalf of the GEF (Global
Environment Facility) partnership.

Which of the statement(s) given above


is/are correct?

a. 1 only
b. 2 only
c. Both 1 and 2
d. Neither 1 nor 2

© IAS.NETWORK
19
Q78) Match the following
Q80) e-AUSHADHI portal which
1. Sela pass – a. Uttarakhand intended for increased transparency
2. Khardung La – b. Arunachal was launched recently by which of the
Pradesh following agencies?
3. Sin La – c. Jammu and Kashmir
a. Ministry of AYUSH
Select the correct answer using the b. Ministry of Health and Family
code given below Welfare
c. Ministry of Chemicals and Fertilizers
a. 1-a; 2-b; 3-c d. Government of Haryana
b. 1-c; 2-a; 3-b
c. 1-a; 2-c; 3-b Q81) Consider the following
d. 1-b; 2-c; 3-a statements with respect to National
Security Act (NSA)
Q79) Consider the following
statements with respect to National 1. It allows preventive detention for
Commission for Safai Karamcharis months, if authorities are satisfied that
(NCSK) a person is a threat to national
security or law and order.
1. It is a statutory body. 2. It applies to the entirety of India,
2. It works under the Ministry of Social except Jammu and Kashmir.
Justice and Empowerment.
3. NCSK is mandated to monitor the Which of the statement(s) given above
implementation of the Prohibition of is/are correct?
employment as manual scavengers
and their rehabilitation Act, 2013. a. 1 only
b. 2 only
Which of the statement(s) given above c. Both 1 and 2
is/are correct? d. Neither 1 nor 2

a. 1 only
b. 1 and 3 only
c. 2 and 3 only
d. 1, 2 and 3

© IAS.NETWORK
20
Q82) Global Fund sometimes seen in Q84) Consider the following
the news recently, is a partnership statements with respect to National
organization, designed to accelerate Board for Wildlife (NBWL)
the end of which of the following as
epidemics? 1. The standing committee of the
NBWL is chaired by the Union minister
a. AIDS for Environment and Forest.
b. Tuberculosis 2. In India, NBWL is responsible for
c. Malaria allowing forest land in protected areas
d. All of the above to be diverted for industries.

Q83) Consider the following Which of the statement(s) given above


statements with respect to Small is/are correct?
Farmers Agribusiness Consortium
(SFAC) a. 1 only
b. 2 only
1. It is an autonomous society working c. Both 1 and 2
under the administrative control of d. Neither 1 nor 2
Ministry of Agriculture & Farmers
Welfare. Q85) “Pashtuns” sometimes seen in
2. It is mandated to mobilize farmers the news recently, inhabits which of
into groups, called Farmers Producers the following countries?
Organisations (FPOs) and link these
institutions to the market, for better a. Syria
returns. b. Israel
c. Azerbaijan
Which of the statement(s) given above d. None of the above
is/are correct?

a. 1 only
b. 2 only
c. Both 1 and 2
d. Neither 1 nor 2

© IAS.NETWORK
21
Q89) Consider the following
Q86) Anoop Satpathy committee statements with respect to Sambar
sometimes seen in the news recently, deer
was constituted to review and
recommend which of the following? 1. It is large in size and endemic to
Indian subcontinent.
a. Methodology for fixation of National 2. It has been protected under
Minimum Wage schedule I of the Wildlife Protection
b. Measures to reorient disinvestment Act, 1972.
policy of India
c. Suggestions to avoid future bank Which of the statement(s) given above
frauds in India is/are correct?
d. Guidelines to combat reducing
glacier cover over Himalayas a. 1 only
b. 2 only
Q87) “World Employment and Social c. Both 1 and 2
Outlook” is an annual report published d. Neither 1 nor 2
by which of the following
organisations? Q90) Consider the following pairs

a. World Bank 1. Pahari Dam – Madhya Pradesh


b. International Labour Organisation c. 2. Kawal Tiger Reserve – Andhra
International Organisation of Pradesh
Employers
d. Oxfam International Which of the pair(s) given above
is/are correctly matched?
Q88) Spirit and Opportunity rovers
sometimes seen in the news recently, a. 1 only
are associated with which of the b. 2 only
following? c. Both 1 and 2
d. Neither 1 nor 2
a. NASA and MOON
b. ESA and JUPITER
c. JAXA and RYUGU
d. NASA and MARS

© IAS.NETWORK
22
Q93) Consider the following
Q91) Consider the following statements with respect to
statements with respect to Right to International Court of Justice (ICJ)
Information Act, 2005
1. It is an intergovernmental
1. It mandates that, a Chief Information International tribunal.
Commissioner should be a person 2. It is located in Hague, Netherlands.
belonging to Public service, of any 3. India is not a party to the ICJ
Government for more than 20 years. statute.
2. Matters related to Collegium and
judicial appointments of Supreme Which of the statement(s) given above
Court comes within the ambit of Right is/are correct?
to Information Act, 2005.
a. 2 only
Which of the statement(s) given above b. 1 and 3 only
is/are correct? c. 1, 2 and 3
d. None
a. 1 only
b. 2 only Q94) Consider the following
c. Both 1 and 2 statements with respect to Black
d. Neither 1 nor 2 Bucks

Q92) In which of the following 1. Normally, female blackbucks do not


conditions, corals cannot survive? possess horns.
2. It has been listed as least
1. No Sunlight concerned in IUCN Red list.
2. Fresh water 3. Stray dogs and barbed wires are
3. Estuaries threatening its survival.
4. Cold water
Which of the statement(s) given above
Select the correct answer using the is/are correct?
code given below
a. 1 only
a. 2 and 3 only b. 1 and 3 only
b. 2 and 4 only c. 2 and 3 only
c. 1, 2 and 4 only d. 1, 2 and 3
d. 1, 2, 3 and 4

© IAS.NETWORK
23
Q97) Abohar Wildlife Sanctuary
(AWS) sometimes seen in the news
Q95) Which one of the following best recently is located in which of the
describes the term “Glacial Lakes” following states?
sometimes seen in news recently?
a. Punjab
a. They are formed when a glacier b. Uttar Pradesh
erodes the land, and then melts within c. Rajasthan
the hole it has created d. Telangana
b. They are formed when a wide
meander from the main stem of a
glacial river is cut off, creating a
free-standing body of water
c. They are formed when water from
the melted glaciers gets stored in a
depressed land surface Q98) Consider the following
d. They are formed due to a difference statements with respect to Kelps
in temperature, resulting in creation of
isolated ice blocks over water bodies 1. Kelps are plants that look similar to
brown algae.
2. They thrive in cold, nutrient-rich
waters.

Which of the statement(s) given above


Q96) SPHEREx mission sometimes is/are correct?
seen in the news recently is _______
a. 1 only
a. NASA‘s Space Telescope b. 2 only
b. ISS‘s airborne Earth Observatory c. c. Both 1 and 2
China‘s mission to study stellar flare d. Neither 1 nor 2
d. None of the above

© IAS.NETWORK
24
Q99) Consider the following
statements with respect to Pangolin

1. It is the most trafficked reptiles in the


world.
2. Indian Pangolin – one among the
eight pangolin species found across
the world is listed as critically
endangered under the IUCN Red list.

Which of the statement(s) given above


is/are correct?

a. 1 only
b. 2 only
c. Both 1 and 2
d. Neither 1 nor 2

Q100) Through which of the following


processes energy can be extracted
from wastes?

1. Incineration
2. Gasification
3. Pyrolysis
4. Anaerobic digestion

Select the correct answer using the


code given below

a. 1 and 3 only
b. 1, 2 and 3 only
c. 2, 3 and 4 only
d. 1, 2, 3 and 4

© IAS.NETWORK
25
ANSWER KEY

1.D 21.B 41.C 61.C 81.C


2.A 22.A 42.D 62.D 82.D
3.B 23.C 43.B 63.C 83.C
4.C 24.A 44.C 64.B 84.C
5.D 25.C 45.D 65.D 85.D
6.C 26.C 46.C 66.C 86.A
7.C 27.D 47.C 67.A 87.B
8.D 28.C 48.C 68.A 88.D
9.B 29.A 49.C 69.D 89.D
10.D 30.A 50.A 70.C 90.D
11.C 31.D 51.D 71.B 91.D
12.C 32.A 52.C 72.B 92.A
13.D 33.A 53.D 73.A 93.A
14.B 34.D 54.C 74.D 94.D
15.A 35.C 55.A 75.A 95.A
16.D 36.D 56.B 76.C 96.A
17.A 37.D 57.B 77.C 97.A
18.B 38.C 58.A 78.D 98.B
19.A 39.D 59.D 79.C 99.D
20.B 40.C 60.D 80.A 100.D

© IAS.NETWORK
26
EXPLANATIONS

Answer 11)

The Defence Acquisitions Council (DAC) has recently approved the


indigenous construction of six submarines worth 40,000 crore rupees under
project 75(India).

It will provide a major boost to submarine design and manufacturing


ecosystem in India through the transfer of design, equipment technology
and skill sets.

Answer 12)

Ministry of Defence‘s ambitious Strategic Partnership (SP) model aims to


provide a significant fillip to the Government‘s „Make in India‟ programme.

● It envisages indigenous manufacturing of major defence platforms by


an Indian Strategic Partner, who will collaborate with foreign OEM to
set up production facilities in the country.
● The model has a long term vision of promoting India as a
manufacturing hub for defence equipment through transfer of niche
technologies and higher Indigenous Content thereby enhancing
self-sufficiency for meeting the future requirements of the Armed
Forces.
● It intends to institutionalise a transparent, objective and functional
mechanism to encourage broader participation of the private sector,
in addition to DPSUs / OFB, in the manufacture of defence platforms
and equipment such as aircraft, submarines, helicopters and
armoured vehicles.
© IAS.NETWORK
27
● The following four segments have been identified for acquisition
under Strategic Partnership (SP) route:
● 1. Fighter Aircraft
● 2. Helicopters
● 3. Submarines
● 4. Armoured Fighting Vehicles (AFVs) / Main Battle Tanks (MBTs).

Answer 13)*

Under Article 112 of the Constitution, a Statement of Receipts and


Payments (estimated) has to be tabled in the Parliament for every financial
year.

● If the Budget is not passed within the announced date, Article 116 of
the Constitution empowers the Lok Sabha to pass the
Vote-OnAccount, a document which covers only the expenditure
incurred.
● A ―vote-on-account‖ only deals with the expenditure in
Government‟s budget, while an ―interim budget‖ includes both
expenditure and receipts.

Answer 14)

It is headquartered in Patancheru (Hyderabad, Telangana, India).

Answer 15)

In June 2011, government of India had announced setting up a


high-powered task force headed by Naresh Chandra, to review the defense
management in the country and make suggestions for implementation of
major defense projects.

Answer 16)
© IAS.NETWORK
28
The Science and Engineering Research Board (SERB), a statutory body of
the Department of Science and Technology (DST) launched the ‗Visiting
Advanced Joint Research (VAJRA) Faculty Scheme‘.
● It aims to connect the Indian academic and research and
development (R&D) systems to the best of global science and
scientists for a sustained international collaborative research.
● It is a dedicated program exclusively for overseas scientists and
academicians with emphasis on Non-resident Indians (NRI) and
Persons of Indian Origin (PIO) / Overseas Citizen of India (OCI) to
work as adjunct / visiting faculty for a specific period of time in Indian
Public funded academic and research institutions.

Answer 17)

The United Nations Security Council has recently adopted a resolution to


extend sanctions against the Central African Republic (CAR) for another
year.
● CAR is a landlocked country in Central Africa.
● It is bordered by Chad to the north, Sudan to the northeast, South
Sudan to the east, the Democratic Republic of the Congo to the
south, the Republic of the Congo to the southwest and Cameroon to
the west.

Answer 18)

Under this programme, vulnerable landholding farmer families, having


cultivable land up to 2 hectares, will be provided direct income support at
the rate of Rs. 6,000 per year.

© IAS.NETWORK
29
● This income support will be transferred directly into the bank
accounts of beneficiary farmers, in three equal instalments of Rs.
2,000 each.
● This programme will be funded by Government of India.
● Around 12 crore small and marginal farmer families are expected to
benefit from this.

Answer 19)

Germany, France and the UK has set up a payment channel with Iran
called INSTEX, to help continue trade and circumvent US sanctions.

Answer 20)

An 11-year-old has become the first patient to receive CAR-T therapy


(immunotherapy) that uses the body's own cells to fight cancer.

● CAR-T is a personalized form of cancer treatment.


● CAR-T involves removing immune cells and modifying them in a
laboratory so they can recognize cancer cells.
● Immunotherapy is treatment that uses your body's own immune
system to help fight cancer.
● First, the patient has blood removed and the white blood cells are
separated out, with the rest of the blood being returned to the patient.
● A harmless virus is used to insert genes into Tcells, a special type of
immune cell.
● These genes cause the T-cells to add a hook on to their surface,
known as a chimeric antigen receptor (CAR).
● These engineered CAR-T cells - programmed to recognize and
destroy the patient's cancer cells - are multiplied in huge numbers
and then infused back into the patient.

Answer 21)
© IAS.NETWORK
30
The world‘s first ship-mounted electromagnetic railgun which is developed
by the Chinese Navy and can strike targets more than 200 km away at
seven times the speed of sound was tested in January 2018 and will be
ready for deployment by 2025.

● It use electricity instead of gunpowder to launch projectiles, and


therefore, eliminates the need to store explosives aboard warships.
● Railguns have long been touted as one of the future technologies of
warfare, with the ability to launch projectiles at Mach 6 - or, more than
4,500 miles per hour.
● The Chinese railgun is believed to be capable of launching projectiles
at nearly 5,600 miles per hour.
● Because the missile is fired using kinetic energy, this eliminates the
risks associated with keeping explosives on a ship.
● U.S had a similar project until last year but scrapped its programme
citing concerns about the colossal amount of power required to fire
them.

Answer 22)

Government of India in its recent interim budget proposes to launch a mega


pension yojana namely 'Pradhan Mantri ShramYogi Maandhan' for the
unorganised sector workers with monthly income upto Rs. 15,000.

● This pension yojana shall provide them an assured monthly pension


of Rs. 3,000 from the age of 60 years on a monthly contribution of a
small affordable amount during their working age.
● The Government will deposit equal matching share in the pension
account of the worker every month.

Answer 23)

© IAS.NETWORK
31
Fiscal deficit for 2019-20 has been pegged at 3.4% of GDP.

● Target of 3% of fiscal deficit to be achieved by 2020-21.


● Fiscal deficit brought down to 3.4% in 2018-19 RE from almost 6%
seven years ago.

Answer 24)

The GoI will setup a Committee under NITI Aayog to complete the task of
identifying Denotified, Nomadic and Semi-Nomadic communities not yet
formally classified.

● The Nomadic and Semi-Nomadic communities move from place to


place in search of a livelihood.
● These communities are hard to reach, less visible, and therefore,
frequently left-out.
● The Renke Commission and the Idate Commission has done
commendable work to identify and list these communities.

Answer 25)

Rashtriya Kamdhenu Aayog aims to upscale sustainable genetic


upgradation of cow resources and to enhance production and productivity
of cows.

● The Aayog will also look after effective implementation of laws and
welfare schemes for cows.

Answer 26)

The Budget for the year 2019-20 has increased the allocation for Rashtriya
Gokul Mission to Rs. 750 crore.

© IAS.NETWORK
32
● It is a scheme, run by the Ministry of Agriculture & Farmers Welfare
aims to conserve and develop indigenous breeds of cattle.

Answer 27)

Know My India Programme is a unique programme initiated by the National


Foundation for Communal Harmony (NFCH) to bring together financially
assisted children of the Foundation from different States/Regions of the
country to promote oneness, fraternity and national integration.

● The NFCH is an autonomous organization under the administrative


control of the Ministry of Home Affairs.
● The main objective of the Foundation is to provide assistance to the
children / youth rendered orphan / destitute in communal, caste,
ethnic or terrorist violence for their rehabilitation besides promoting
communal harmony and national integration through various
activities.
● Know India Programme (KIP) is a flagship initiative for Diaspora
engagement which familiarizes Indian-origin youth (18-30 years) with
their Indian roots and contemporary India, through a three-week
orientation programme organised by the Ministry of External affairs.

Answer 28)

Bharat Rang Mahotsav (BRM), the annual international theatre festival of


India is organized by the National School of Drama (NSD), was established
two decades ago to stimulate the growth and development of theatre
across India.

● Originally a national festival showcasing the work of the most creative


theatre workers in India, BRM has evolved to international scope,
hosting theatre groups from around the world, and is now the largest
theatre festival of Asia.
© IAS.NETWORK
33
● 20th Bharat Rang Mahotsav was recently inaugurated in New Delhi.

Answer 29)

The black softshell turtle (Nilssonia nigricans) figures in the International


Union for Conservation of Nature‘s (IUCN) Red List as ―extinct in the
wild‖.
● But a few temple ponds in Assam and Bangladesh are bringing these
turtles back from the brink.
● Being received the divine protection for many years, the rarest of
India‘s 28 turtle species is back where it belongs (in the wild).
● Recently, 35 hand-reared turtle hatchlings, including 16 black
softshells, in the Haduk Beel (wetland) of Pobitora Wildlife Sanctuary.

Answer 30)

Through budget 2019-20, Government eMarketplace (GeM) is now being


extended to all CPSEs.

Answer 31)

The austere, grandiose site of Hampi was the last capital of the last great
Hindu Kingdom of Vijayanagar.

● Group of Monuments at Hampi, a UNESCO World Heritage Site is


located in the Tungabhadra basin in Bellary District, Central
Karnataka.

Answer 32)

© IAS.NETWORK
34
Battle of Chausa was fought between Humayun and Sher Shah Suri in
1539.

● The Mughal forces under Humayun were defeated by Sher Shah.


Humayun narrowly escaped.
● Battle of Kannauj in which, Mughal forces once again clashed with
Sher Shah in May 1540.
● This time Humayun was decisively defeated by Sher Shah.
● Humayun left India and went into exile. He remained in exile for the
next 15 years.

Answer 33)

Recently, 35 hand-reared turtle hatchlings, including 16 black softshells, in


the Haduk Beel (wetland) of Pobitora Wildlife Sanctuary.

Answer 34)

The Government has recently unveiled its vision for the next decade, listing
thereby ten most important dimensions in 2030.

● This comprehensive ten-dimensional Vision will create an India where


poverty, malnutrition, littering and illiteracy would be a matter of the
past.
● The vision was presented by the Indian Finance Minister during his
Budget speech recently in Parliament.

Answer 35)

The RBI has specified certain regulatory trigger points, as a part of PCA
Framework for initiation of certain structured and discretionary actions in
respect of banks hitting such trigger points.

© IAS.NETWORK
35
● Under the revised PCA framework: Capital, Asset quality and
Profitability continue to be the key areas for monitoring.
● Indicators to be tracked for Capital, asset quality and profitability
would be CRAR/ Common Equity Tier I ratio, Net NPA ratio (NNPA)
and Return on Assets (RoA) respectively.
● Leverage would be monitored additionally as part of the PCA
framework.
● The PCA framework is applicable only to commercial banks and not
extended to cooperative banks, non-banking financial companies
(NBFCs) and FMIs.

Answer 36) The National Sample Survey Office‟s Periodic Labour Force
Survey showed that India‘s labour force participation rate, which is the
proportion of the population working or seeking jobs, fell to 49.8% in 2017-
‘18 from 55.9% in 2011-‘12.

Answer 37) At the heart of the dispute between the China and Japan are
eight uninhabited islands and rocks in the East China Sea.
● The Japanese call them Senkaku, and designate them as sovereign
territory.
● On the contrary, China also claimed these islands and named them
as Diaoyu Islands.
● Currently, these Islands are administered by Japan.
● Recently, large ships of the Japanese Coast Guard keep a tight
round- the-clock vigil on this group of Islands.
● Their mission is to ensure that Japan maintains its hold over these
small but strategically significant islands in the East China Sea.

Answer 38) C

Answer 39) Recently, PM has laid the foundation stone of 624 MW Kiru
Hydroelectric project in Kishtwar, Jammu Kashmir.

© IAS.NETWORK
36
● It is a run of river project across River Chenab and once completed,
the project will generate 2272 Million Units of electricity annually.
● PM has also inaugurated the 9 MW Dah Hydroelectric project.
● Located in Dah near village Datang, this project is a run-of–the-river
scheme, by Jammu & Kashmir State Power Development
Corporation Limited.
● Tremors of light intensity were experienced in villages in the vicinity of
Varahi hydroelectric project in Hosanagar and Thirthahalli taluks of
Shivamogga district, Karnataka.

Answer 40) Department of School Education and Literacy has launched an


Integrated Scheme for School Education called Samagra Shiksha Abhiyan.

● It subsumes three erstwhile Centrally Sponsored Schemes of Sarva


Shiksha Abhiyan (SSA), Rashtriya Madhyamik Shiksha Abhiyan
(RMSA) and Teacher Education (TE).
● The new integrated scheme envisages school education as a
continuum from pre-school to senior secondary level.

Answer 41)

The Ministry of Social Justice and Empowerment has introduced a scheme


for providing physical aids and Assisted Living Devices for Senior Citizens
belonging to BPL category named ―Rashtriya Vayoshri Yojana (RVY)‖
on 1st April, 2017.

● It provides Senior Citizens, belonging to BPL category and suffering


from age related disabilities/ infirmities, with such physical aids and
assisted living devices which can restore near normalcy in their bodily
functions.
● The Scheme is being implemented through the ―Artificial Limbs
Manufacturing Corporation (ALIMCO)‖, a Public Sector Undertaking
under the Ministry of SJ&E, as the sole Implementing Agency.
© IAS.NETWORK
37
Answer 42)

Recently, the Ministry of Labour has lifted the time bar on the employment
of women in mines across the country.

● Previously Section 46 of the Mines Act, 1952 prohibited the


employment of any women in any part of a mine which is
below-ground and on ground, except between 6 a.m. and 7 p.m.

Answer 43)

Pravasi Kaushal Vikas Yojana (PKVY) is aimed at providing suitable skill


sets for those seeking employment abroad by training them in different
trades and also imparting soft skills.

● PKVY is a skill development initiative by the Ministry of External


Affairs.
● The MEA and the Ministry of Skill Development and Entrepreneurship
(MSDE) had signed a Memorandum of Understanding (MoU) for its
implementation.
● The scheme will be implemented by National Skill Development
Corporation (NSDC) through its training partners in consultation with
Ministry of External Affairs and Ministry of Skill Development and
Entrepreneurship.

Answer 44) Earthquake swarm is a series of many (sometimes thousands)


low magnitude earthquakes without a discernible main shock.

● They occur in a localised region and over a period of time ranging


from days, weeks to even months, without a clear sequence of
foreshocks, main quakes and aftershocks.
© IAS.NETWORK
38
● When seismic energy piles up inside the Earth and is released in
small amounts from certain points, such a series of earthquakes can
occur.
● Sometimes, these are also accompanied by acoustic or sound
emissions.
● Dahanu town in Maharashtra‟s Palghar district has been hit by some
30 low-intensity earthquakes since November 2018.
● Palghar was rocked by a series of minor earthquakes (6 tremors) on
a single day recently.
● The magnitudes of the quakes ranged between 3 and 4.1 on the
Richter scale.
● Many aftershocks of lesser magnitude have also been observed in
the area.
● Data collected so far point to an “earthquake swarm”.

Answer 45)

The Global Crisis of Nuclear Waste is a report published by Greenpeace


France in January 2019.

● An analysis of waste storage facilities in seven countries with nuclear


power revealed that several were near saturation.

Answer 46)

The Cabinet Committee of Economic Affairs recently gave its approval for
the creation of a corpus of Rs. 2000 crore for Agri-Market Infrastructure
Fund (AMIF) to be created with NABARD for development and upgradation
of agricultural marketing infrastructure in Gramin Agricultural Markets and
Regulated Wholesale Markets.

Answer 47)
© IAS.NETWORK
39
The North Atlantic Treaty Organization (NATO) is an alliance of 29
countries from North America and Europe committed to fulfilling the goals
of the North Atlantic Treaty signed on 4 April 1949.

● In accordance with the Treaty, the fundamental role of NATO is to


safeguard the freedom and security of its member countries by
political and military means.
● Macedonia has recently signed an accord to join NATO to become
the 30th member of the US-led alliance.
● The Macedonia-NATO accord follows a deal with Greece ending a
27-year-old dispute over Macedonia's name.
● The accord must now be ratified by allied governments.

Answer 48) C

Answer 49)

At present, bulk of the incentives to exporters is under the popular MEIS


wherein the government gives incentives to exporters‘ equivalent to a
certain percentage of their export value in the form of duty credit scrips that
can be used to pay customs duties and are freely transferable.

● But with the WTO now ruling that since India‘s per capita Gross
National Income is over $1,000 it is no longer eligible to give direct
subsidies such as the ones offered under MEIS, such schemes have
to be phased out.
● Since the government is not keen on giving more direct export
subsidies such as the one given under the MEIS, the ROSL becomes
more relevant.

© IAS.NETWORK
40
Answer 50)

The National Minorities Development and Finance Corporation (NMDFC) is


a not for profit company under the companies act 1956.

● It works under the aegis of Ministry of Minority Affairs.


● It is a National Level Apex Body for the benefit of Minorities as
defined under the National Commission for Minorities Act 1992.
● The prime mandate of NMDFC is to provide concessional finance to
the Minorities for selfemployment/ income generation activities.
● The notified Minorities in India are Muslims, Christians, Sikhs,
Buddhists & Parsis and Jain.
● Under NMDFC program, preference is given to Artisans & Women.

Answer 51)

India has climbed eight places to rank 36th in the latest annual
International IP Index released by the US Chambers of Commerce.

● In 2018 International Intellectual Property Index, India was ranked 44


out of 50 countries.

Answer 52)

The CBI, which functions under the provisions of the Delhi Special Police
Establishment (DSPE) Act, 1946, can probe offences in a state with prior
approval of the state government concerned.

© IAS.NETWORK
41
● Further, Constitutional courts can also entrust any case or class of
case for investigation in exercise of inherent jurisdiction even without
the consent of the respective state government.
● Further, in the cases which are referred by the Constitutional courts,
the entry of CBI cannot be denied by that state as these do not
require the consent of the state.
● Withdrawal of consent, if any, by a state government can be effected
prospectively and not retrospectively.

Answer 53)

e-NAM (National Agriculture Market) is a panIndian electronic trading


(e-trading) portal which seeks to network the existing physical regulated
wholesale market (known as APMC market) through a virtual platform to
create a unified national market for agricultural commodities.

● Inter State trade of agricultural produce in wholesale mandis through


National Agriculture Market (e-NAM) platform is gathering pace in
recent times.

Answer 54)

Under the INF Treaty, the United States and Soviet Union agreed to ban all
landbased ballistic and cruise missiles with ranges between 500 and 5,500
kilometres.

● The ban applied to missiles with both nuclear and conventional


warheads, but it did not apply to sea-based or air-delivered missiles.

© IAS.NETWORK
42
● The U.S. has recently decided to withdraw from the
Intermediate-Range Nuclear Forces (INF) treaty with Russia.
● The U.S.‘s unilateral withdrawal from a nuclear treaty threatens to
trigger a new arms race.

Answer 55)

Ratoon cropping is a form of cultivation in which a second crop is allowed


to grow from the remains of one already harvested.

● It is also called stubble cropping as the new plants grow from the
stubble of the harvested crop.
● Sugarcane, pineapple and banana are crops in which this method is
practised.
● Ratooning cannot be used endlessly as the yield and quality
decrease after each cycle.
● In sugarcane, for example, two or three ratooning crops are possible.
After which fresh planting has to be done.
● Rice ratooning is also possible if adequate water is available for the
second crop.

Answer 56)

The 13th Conference of Parties (COP) of the Convention on the


conservation of migratory species of wild animals (CMS), an environmental
treaty under the aegis of United Nations Environment Programme, is going
to be hosted by India during 22nd February, 2020 at Gandhinagar in
Gujarat.

● CMS, also referred to as the Bonn Convention provides a global


platform for the conservation and sustainable use of migratory
animals and their habitats and brings together the States through
which migratory animals pass, the Range States, and lays the legal
© IAS.NETWORK
43
foundation for internationally coordinated conservation measures
throughout a migratory range.
● India has been a Party to the CMS since 1983 when the convention
entered into force.
● The Conference of Parties (COP) is the decision-making organ of this
convention.
● India has also signed non legally binding MOU with CMS on the
conservation and management of Siberian Cranes (1998), Marine
Turtles (2007), Dugongs (2008) and Raptors (2016).

Answer 57)

The Indian sub-continent is also part of the major bird flyway network, i.e,
the Central Asian Flyway (CAF) that covers areas between the Arctic and
Indian Oceans, and covers at least 279 populations of 182 migratory water
bird species, including 29 globally threatened species.

● India has also launched the National Action Plan for conservation of
migratory species under the Central Asian Flyway.

Answer 58)

Election Commission of India (ECI) has recently conducted a 2-day


intensive Trainingcum-Workshop on the use of Information &
Communication Technology for the conduct of General Elections 2019.

© IAS.NETWORK
44
● One of the important applications for which training was imparted to
all States and UTs was cVIGIL.
● cVIGIL App provides time-stamped, evidence based proof of Model
Code of Conduct / Expenditure Violation, having live photo/video with
auto location data.
● Any citizen can lodge a complaint through Mobile App.
● Flying Squads then investigate the matter and the Returning Officer
takes the decision.
● ECI has also launched a Voter Verification and Information
Programme (VVIP) for citizens for verifications of their names, new
registrations, and changes in the voter details and corrections in the
Voter Id Cards for the upcoming General Elections 2019 during the
workshop.

Answer 59)

The SFDR technology was jointly developed by India (DRDO) and Russia.

● The ramjet propulsion system used in the SFDR acts as an oxidizer


and the solid propellant reacts as air flows through a solid propellant
duct.
● Unlike conventional rockets that carry propellant and oxidizer, Ramjet
uses the air as an oxidizer just like a jet engine. Therefore the weight
of the fuel required is eliminated.
● Defence Research and Development Organisation (DRDO)
successfully flight tested the second indigenously developed ‗Solid
Fuel Ducted Ramjet (SFDR)‘ propulsion based missile system from
ITR, Chandipur, Odisha recently.
● The success of SFDR propulsion technology will pave the way for the
development of longrange air-to-air missiles in the country.

Answer 60)

© IAS.NETWORK
45
India is temporary home to several migratory animals and birds.

● The important among these include Amur Falcons, Bar headed


Gheese, Black necked cranes, Marine turtles, Dugongs,
Humpbacked Whales, etc.

Answer 61)

HeliNa is an air-to-land version of Nag missile, which is one of the five


missile systems developed by DRDO under the Integrated Guided Missile
Development Programme (IGMDP).

● India, successfully flight tested the indigenously developed and


modified anti-tank missile HeliNa, guided from a defence facility off
Odisha coast paving the way for its actual launch from helicopter.

Answer 62)

Mera aspataal (My Hospital) is a Ministry of Health, Government of India


initiative to capture patient feedback for the services received at the
hospital through user-friendly multiple channels such as Short Message
Service (SMS), Outbound Dialling (OBD) mobile application and web
portal.

● The patient can submit the feedback in seven different languages on


mobile app and web portal; for the hospitals visited in last 7 days.
● It will help the government to take appropriate decisions for
enhancing the quality of healthcare delivery across public facilities
which will improve the patient‘s experience.

Answer 63) C

Answer 64)
© IAS.NETWORK
46
Competition Commission of India is a statutory body of the Government of
India, responsible for enforcing the Competition Act, 2002 throughout India
(except J&K) and to prevent activities that have an adverse effect on
competition.

Answer 65) D

Answer 66)

In ―hydro-seismicity‖ water from heavy rainfall enters small fractures in


rocks, which raises the pressure.

● The pressure built up due to the rise in ground water is released in


earthquake swarms.
● It is estimates that with every 10 meter rise in groundwater pore
pressure increases by 1 bar.

Answer 67) A

Answer 68) Its objective is to give rural people affordable life insurances
services.

Answer 69) D

Answer 70)

The Maharashtra government has recently initiated a plan to clean, restore


and beautify the iconic Gateway of India in South Mumbai.

● The Gateway of India is an arch monument built during the 20th


century.

© IAS.NETWORK
47
● The iconic structure, which overlooks the Arabian Sea, was erected
to commemorate the visit of King George V and Queen Mary at
Apollo Bunder to Mumbai.
● The foundation stone of the monument was laid on March 31, 1913
and its construction was completed in 1924.
● It is built in Indo-Saracenic style.
● India Gate which stands at the centre of New Delhi commemorates
the 70,000 Indian soldiers who lost their lives fighting for the British
Army during the World War I.
● The memorial bears the names of more than 13,516 British and
Indian soldiers killed in the North-western Frontier in the Afghan war
of 1919.

Answer 71)

The World Sustainable Development Summit (WSDS) is the annual


flagship event of The Energy and Resources Institute(TERI).

● WSDS 2019 will focus on the theme, 'Attaining the 2030 Agenda:
Delivering on our Promise'.
● The ―mega themes‖ to be addressed during the event are clean
oceans, climate finance, energy transitions, mobility and sustainable
agriculture.

Answer 72)

Recently, Prime Minister of India has declared that a ‗Blue Flame


Revolution‘ is under-way.

● LPG coverage has reached more than 90% percent, from 55% five
years ago.

© IAS.NETWORK
48
● The Union Minister for petroleum and Natural gas has recently stated
that, ―In addition to the general development of the hydrocarbon
industry, we have also focused on improving the ease of living of the
common people by enhancing access to clean cooking fuel which we
have called the Blue Flame Revolution‖.

Answer 73)

Every Finance Commission has a special theme. ‗

● GST Restructuring‘ was the special theme of the Thirteenth Finance


Commission headed by Shri Vijay L. Kelkar.
● Health and Education would be a special theme of the Fifteenth
Finance Commission headed by Shri Nand Kishore Singh.

Answer 74)

The e-Office is a Mission Mode Project under the National e-Governance


Plan and is being implemented by the Department of Administrative
Reforms and Public Grievances (DARPG) to improve efficiency in
Government Process and Service Delivery Mechanism.

● The e-Office is a Digital Workplace Solution that replaces the physical


files and documents with an efficient electronic system.

Answer 75)

Dard Aryans Tribes inhabit Dha, Hanu, Beema, Darchik and Garkone
villages in Leh and Kargil districts and are together called the Aryan valley.

● The people of this region have unique physical features, social life,
ethnic culture and language.

© IAS.NETWORK
49
● Researchers believe that the ‗Aryans of Ladakh‘ or the ‗Brokpas‘ are
descendants of Alexander‘s army and had come to the region over
2,000 years ago.
● A delegation of the Dard Aryans recently submitted their charter of
demands to Minister of State for Tribal Affairs.

Answer 76) C

Answer 77)

Small Grants Program (SGP) is implemented by UNDP on behalf of the


GEF (Global Environment Facility) partnership.

● SGP provides financial and technical support to communities and


Civil Society Organizations to meet the overall objective of global
environmental benefits secured through community-based initiatives
and actions.
● Through a decentralized, national-level delivery mechanism, SGP
finances community-led initiatives to address global environmental
issues.
● The Program is specifically designed to mobilize bottom-up actions
by empowering local civil society organizations, and poor and
vulnerable communities, including women and Indigenous Peoples.

Answer 78)

© IAS.NETWORK
50
The foundation stone for Sela Tunnel Project was laid recently.

● Sela Pass also called as Se La is the highaltitude mountain pass


located in Tawang District of Arunachal Pradesh.
● It has an elevation of 4170 m (13,700 ft) connects the Buddhist city of
Tawang Town to Tezpur and Guwahati.
● It is the main road connecting Tawang with the rest of India.
● The Sela Lake also known as Paradise Lake is located near to the
pass.
● It is usually open throughout the year unless landslides or snow
require the pass to be shut down temporarily.

Answer 79)

The National Commission for Safai Karamcharis (NCSK) was established


as a statutory body in the year 1994 as per the provisions of the NCSK Act
1993 initially for the period of 3 years.

● Later the validity of the Act was initially extended up to 31.3.2002 and
thereafter up to 29.2.2004.
● The NCSK Act ceased to have effect from 29.2.2004.
● After that the tenure of the NCSK has been extended as a
non-statutory body from time to time.
● The tenure of the present Commission is up to 31.3.2019.
● The Union Cabinet has recently approved the proposal for Extension
of tenure of the National Commission for Safai Karmacharis (NCSK)
beyond 31.3.2019 for three years.

Answer 80)

To increase transparency and accountability, the Ministry of AYUSH has


recently launched the e-AUSHADHI portal for online licensing of Ayurveda,
Siddha, Unani and Homoeopathy drugs and related matters.
© IAS.NETWORK
51
● AUSHADHI was an acronym for Ayurveda, Unani, Siddha and
Homeopathy Automated Drug Help Initiative.

Answer 81)

The National Security Act is a stringent law that allows preventive detention
for months, if authorities are satisfied that a person is a threat to national
security or law and order.

● The person does not need to be charged during this period of


detention.
● The goal is to prevent the individual from committing a crime.
● It applies to the entirety of India, except Jammu and Kashmir.

Answer 82)

The Global Fund is a 21st-century partnership organization designed to


accelerate the end of AIDS, tuberculosis and malariaas epidemics.

● Founded in 2002, the Global Fund is a partnership between


governments, civil society, the private sector and people affected by
the diseases.
● India is in a sustained partnership with the Global Fund to fight
against AIDS, Tuberculosis and Malaria (GFATM) since 2002 as
recipient as well as donor.
● The Global Fund raises funds in multiyear cycles known as
Replenishments and is preparing for sixth voluntary replenishment
conference to mobilize needed resources to scale up life-saving
programs over 2020-22.
© IAS.NETWORK
52
● A high level preparatory meeting for the sixth replenishment
conference of the Global Fund is hosted by India recently in New
Delhi.

Answer 83)

SFAC is a registered society set up under the Societies Registration Act


XXI of 1860 under the administrative control of Department of Agriculture
Cooperation and Farmers Welfare, Ministry of Agriculture & Farmers
Welfare.

● Its mandate is to link farmers to investment, technology and markets.


● SFAC is mandated by the M/o Agriculture to mobilize farmers into
groups, called Farmers Producers Organisations (FPOs) and link
these institutions to the market, for better returns.
● These bodies are member owner grass root institutions, which
provide collective bargaining powers to small and marginal farmers.
● SFAC proposes to establish a Kisan Mandi – as an additional
marketing channel/platform to link FPOs and farmers Growers
Association to wholesale and retail consumers for direct sale of fruits
& vegetables for the benefits of farmers and consumers in Delhi &
NCR.

Answer 84)

© IAS.NETWORK
53
National Board for Wildlife (NBWL) is a statutory Board constituted in 2003
under Section 5 of the Wild Life (Protection) Act, 1972.

● The NBWL is chaired by the Hon‘ble Prime Minister.


● The Standing Committee of the NBWL is chaired by Union
Environment Minister.
● A Standing Committee of the NBWL is charged with deliberating on
the merits of projects that come to it for scrutiny.
● National Board for Wildlife (NBWL) has been charged with allowing
forest land in Protected Areas to be diverted for industry.

Answer 85)

Pashtuns mainly live in Pakistan and Afghanistan.

● Pashtun people are Afghanistan's largest ethnic group and are also
the second-largest ethnicity in Pakistan.
● Pashtuns are united by the Pashto language, which is a member of
the IndoIranian language family, although many also speak Dari
(Persian) or Urdu.
● They are also known as "Pathans".

Answer 86)

The Ministry of Labour and Employment had constituted an expert


committee, under the Chairmanship Dr. Anoop Satpathyto review and
recommend methodology for fixation of National Minimum Wage (NMW).
© IAS.NETWORK
54
● The Expert Committee has submitted its report recently.

Answer 87)

World Employment and Social Outlook is an annual report published by


International Labour Organisation (ILO), a UN agency.

● Global unemployment levels fell one percentile to a steady 5 per cent


in 2018, the lowest figures since the economic crisis that wreaked
havoc on labour markets, the International Labour Organization said
in its recent report World Employment and Social Outlook Trends
2019.

Answer 88)

Spirit and Opportunity were identical, golf-cart-sized, solar-powered rovers.

● Spirit landed at Gusev Crater on January 4, 2004; Opportunity


followed, landing on the opposite side of Mars at Meridiani Planum on
January 24.
● Contact with Spirit was lost in March 2010, and the mission was
declared over on May 25, 2011.
© IAS.NETWORK
55
● Opportunity continued to roam Mars — and sent out its last signals
on June 10, 2018, when the most intense dust storm in recorded
Martian history encrusted its solar panels, and damaged critical
components.
● NASA has recently announced the end of the Opportunity rover‟s
mission.

Answer 89)

The sambar deer is a large deer native to the Indian subcontinent, southern
China, and Southeast Asia.

● It has been listed as Vulnerable on the IUCN Red List.


● Sambar is a protected animal under schedule three of Wildlife
Protection Act, 1972.

Answer 90) D

Answer 91)

According to RTI Act, 2005, the Chief Information Commissioner and


Information Commissioners shall be persons of eminence in public life with
wide knowledge and experience in law, science and technology, social
service, management, journalism, mass media or administration and
governance.

● In its recent judgment the SC had stated that “official bias” in favour
of bureaucrats and government employees was evident from the very
beginning of the process for appointment of Chief Information
Commissioners and Information Commissioners.
● In fact, the selection committee, which shortlists candidates for
appointment, is itself composed of government employees.

© IAS.NETWORK
56
● The apex court directed the government to look beyond bureaucrats
and appoint professionals from “all walks of life”.
● The Supreme Court also lauded the role of the Right to Information
(RTI) Act as an ―integral part of any vibrant democracy.‖
● But the apex court itself has refused to come within the ambit of the
information transparency law for the past one decade.
● The court has firmly resisted back-to-back decisions of the Central
Information Commission (CIC) and the Delhi High Court to open up to
the RTI regime as far as the issues of Collegium and judicial
appointments are concerned.
● In fact, the appeal filed by the Supreme Court against public
disclosure under RTI has been pending in the Supreme Court since
2010

Answer 92)

Deep sea corals, also known as cold-water corals don‟t require Sunlight.

● Corals need saltwater to survive and require a certain balance in the


ratio of salt to water.
● This is why corals don‘t live in areas where rivers drain fresh water
into the ocean (―estuaries‖).
● Highly polluted dirty water from a flood crisis in northern Australia has
spread to parts of the Great Barrier Reef, placing it under severe
stress.
© IAS.NETWORK
57
● Scientists warned that this could starve corals of light and provide
fodder for the predatory crown-of-thorns starfish.

Answer 93)

International Court of Justice is the principal judicial organ of the United


Nations (UN).

● It was established by the United Nations Charter, which was signed in


1945 in San Francisco (United States), and began work in 1946 in the
Peace Palace, The Hague (Netherlands).
● All Members of the United Nations are ipso facto parties to the
Statute of the International Court of Justice.

Answer 94)

Blackbuck hunting and poaching are prohibited in India as per Schedule I


of the Wildlife Protection Act of 1972.

● Blackbucks used to cover the entire subcontinent of India except for


the Western coast, but as the human population kept growing, their
numbers began plummeting.
● The most striking feature of blackbucks is the long, spiralling horns of
the adult male which have ridges from the base to almost the tips.
Females do not possess horns.
● Stray cattle and dogs, barbed „cobra‟ wire and changed land use
patterns spell trouble for the majestic Punjab‘s State animal: Black
Buck, which has been listed as least concern under the IUCN Red
list.

Answer 95)

© IAS.NETWORK
58
The most worrying outcome of glacial shrinking and retreat is the increase
in the formation and size of glacial lakes.

● Glacial lakes are formed when a glacier erodes the land, and then
melts within the hole it has created.
● This in turn could lead to glacial lake outburst floods (GLOF) when
there is an avalanche or an earthquake.
● Satellite data shows that the number of such lakes in the Hindu
Kush-Himalayan region grew to 4,260 in a decade from 3,350 in
1990.

Answer 96)

NASA will launch a new space telescope in 2023 that could provide a
glimpse of the first moments in the history of the universe, and explore how
common are the ingredients for life in our galaxy‘s planetary systems.

● The Spectro-Photometer for the History of the Universe, Epoch of


Reionization and Ices Explorer (SPHEREx) mission is a planned
two-year mission funded at USD 242 million, according to NASA.
● SPHEREx will survey the sky in optical as well as near-infrared light
which, though not visible to the human eye, serves as a powerful tool
for answering cosmic questions.

Answer 97)

In 2019, so far, as many as eight blackbucks have died in the Abohar


Wildlife Sanctuary (AWS), and a majority of them succumbed to their
injuries, caused by barbed wires while trying to escape from stray dogs.
© IAS.NETWORK
59
Answer 98)

Kelp Forests are underwater ecosystems formed in shallow water by the


dense growth of several different species known as kelps.

● Kelps are actually extremely large brown algae, although they look
like plants.
● They thrive in cold, nutrient-rich waters.
● Kelp attaches to the seafloor and eventually grows to the water‘s
surface and relies on sunlight to generate food and energy.
● Kelps live further from the tropics than coral reefs, mangrove forests,
and warm-water seagrass beds, so kelp forests do not overlap with
those systems.
● Climate change could lead to declines of underwater kelp forests
through impacts on their micro biome.
● It was predicted that ocean warming and acidification can change
microbes on the kelp surface, leading to disease and potentially
putting fisheries at risk.

Answer 99)

Though many think of them as reptiles, pangolins are actually mammals.

● They are the most trafficked mammal in the world.


● There are eight pangolin species – four each in Africa and Asia (two
of them are in India).
© IAS.NETWORK
60
● Indian Pangolin one among the eight pangolin species is listed under
the endangered category of IUCN Red list.
● Obsession for its supposedly medicinal scales in China is believed to
have made the anteating Chinese Pangolin, one of the two species
found in South Asia, extinct in India.

Answer 100)

Waste to Energy (WTE), is a term that is used to describe various


technologies that convert non-recyclable waste into usable forms of energy
including heat, fuels and electricity.

● WTE can occur through a number of processes such as incineration,


gasification, pyrolysis, anaerobic digestion, and landfill gas recovery.

© IAS.NETWORK
61
IAS.NETWORK

PRELIMS 2020 MENTORSHIP PROGRAM

( SEPTEMBER 2019 ) COMPILATION

COVERS ALL MAJOR NEWSPAPERS AND MAGAZINES

© IAS.NETWORK
1
Automatic Exchange of​ ​Information (AEOI) Regime

Automatic Exchange of Information (AEOI) is systematic and periodic transmission of


“bulk” taxpayer information by the source country to the residence country, which is
possible under most of the Double Taxation Avoidance Agreements (DTAAs) and
Multilateral Convention on Mutual Administrative Assistance in Tax Matters (MAC).

-It aims to reduce global tax evasion. It is to be carried out under Common Reporting
Standard (CRS) of OECD.AEOI is the exchange of information between countries
without having to request it.

India's first garbage cafe in Chhattisgarh

-Under this, the Municipal Corporation will provide food to the poor and homeless in lieu
of plastic waste

Namaste Pacific' - Culture of countries in Pacific region showcased in Delhi

A cultural event called ‘Namaste Pacific’ to showcase in India the culture of Pacific
countries such as Australia, New Zealand, Papua New Guinea and Fiji was held in New
Delhi. The event was jointly organized by High Commissions of Australia, New Zealand,
Papua New Guinea, and Fiji.

Build for Digital India Programme

Google and Union Ministry of Electronics and IT (MeitY) launched ‘Build for Digital India’
programme for engineering students

© IAS.NETWORK
2
Features-

1. The programme will give engineering students a platform to develop


market-ready, technology-based solutions that address key social problems.
2. Applicants will take part in online and offline learning opportunities on key
technologies such as machine learning, cloud and android.
3. These will be offered through Google’s Developer Student Club network and
other Google Developer networks.
4. Google will also offer mentorship sessions in product design, strategy and
technology to the most promising products and prototypes.

Kalimantan: Indonesia to move capital from Jakarta to Borneo Island

Government of Indonesia has announced to move its current capital from Jakarta (on
the island of Java) to the sparsely populated East Kalimantan (on Borneo island).

Why shift-

Indonesia’s current capital, Jakarta, is home to more than 10 million people. It


climate-threatened megacity as it has some of the world’s worst air pollution and traffic
congestion. Poor urban planning has left 40% of the city below sea level. The shifting of
capital city is part of a broader strategy to decentralise Indonesia’s economic growth as
Jakarta is home to 60% of the country’s population and accounts for over half of its
economic activity.

Borneo Island

It is the third-largest island in the world and the largest in Asia. It is home to some of
the world’s greatest tropical rainforests. It is politically divided among three countries:

© IAS.NETWORK
3
Indonesia, Malaysia and Brunei (see map below). Kalimantan is Indonesian portion of
the island of Borneo and it comprises 73% of island’s geographic area.

Interpol Red Notice

Red Notices (RNs)

● Criminals or suspects often flee to other countries to evade facing justice. An RN


alerts police forces across the world about fugitives who are wanted
internationally.
● Interpol describes an RN as “a request to law enforcement worldwide to locate
and provisionally arrest a person pending extradition, surrender, or similar legal
action”.
● RNs contain information that helps identify wanted persons, such as their names,
dates of birth, nationality, and physical attributes such as the colour of their hair
and eyes.
● It also includes pictures and biometric data such as fingerprints, if they are
available.

INTERPOL

● The International Criminal Police Organization (INTERPOL) is an international


organization that facilitates worldwide police cooperation and crime control.
● Headquartered in Lyon, France, it was founded in 1923 as the International
Criminal Police Commission (ICPC).
● NTERPOL provides investigative support, expertise, and training to law
enforcement worldwide in battling three major areas of transnational crime:
terrorism, cybercrime, and organized crime.
● Its broad mandate covers virtually every kind of crime, including crimes against
humanity, child pornography, drug trafficking and production, political corruption,
copyright infringement, and white-collar crime.
● The agency also helps coordinate cooperation among the world’s law
enforcement institutions through criminal databases and communications
networks.

© IAS.NETWORK
4
● India accepted Interpol membership in June 1956.

India’s longest electrified railway tunnel between Cherlopalli and Rapuru


(AP)
The 6.7-km-long tunnel is an “engineering marvel” and said it would provide viable rail
connectivity between the Krishnapatnam Port and the hinterland for the seamless
movement of freight.

Samudrayaan Project

It is a pilot project of the Ministry of Earth Sciences for deep ocean mining for rare
minerals.It proposes to send men into the deep sea in a submersible vehicle for ocean
studies.

The project is expected to become a reality by 2021-22.The project has been


undertaken by the National Institute of Ocean Technology (NIOT).

Tamil Nadu’s Dindigul lock and Kandangi saree gets GI tag

Geographical Indication (GI) tag to Dindigul lock and Kandangi saree from Tamil Nadu.
This takes total number of such indigenous products that have been accorded GI tag
from Tamil Nadu to 31.

Dindigul locks

They are known throughout world for their superior quality and durability. More than
3,125 lock manufacturing units are limited to an area of 5 km in and around Dindigul.
This is the main reason Dindigul city is also called as Lock City.

© IAS.NETWORK
5
Kandangi sarees

They are manufactured in entire Karaikudi taluk in Sivaganga district of Tamil Nadu.
They are made up of high quality cotton from Coimbatore. The main characteristic of
these sarees are its bright colours that hold strong. They are characterised by large
contrast borders.

Alliance for Multilateralism

● It is a German initiative backed by France.


● Goal of the alliance is “to show that the states that support multilateralism and
the United Nations remain the majority.
● It does not include the US, Russia, and China, but is drawing many middle
powers like Japan and Canada in the developed world and South Africa in the
developing world.
● The alliance is set to be launched later this month on the margins of the annual
session of the UN General Assembly in New York.

Libra

Facebook announced launching a cryptocurrency called Libra, designed to appeal to its


global user base of over 2 billion. It will be backed by a basket of fiat currencies. It is
supported by a consortium of large-scale corporate houses, financial services firms, and
venture capitalists.

September month to be celebrated as ‘Rashtriya Poshan Maah’

Union Government is celebrating the entire month of September as “Rashtriya Poshan


Maah” (National Nutrition Month) to emphasise the need for healthy nutrition. This
initiative is aimed at sensitising public on healthy eating, addressing twin issues of

© IAS.NETWORK
6
malnutrition and undernutrition and obesity in some sections and also intensifying
existing nationwide campaign for ‘malnutrition-free India.

POSHAN Abhiyaan

POSHAN stands for Prime Minister’s Overreaching Scheme for Holistic Nourishment.
The POSHAN Abhiyaan is a multi- ministerial convergence mission with vision to
address malnutrition with a targeted approach by 2022.It aims at achieving
improvement in nutritional status of children up to 6 years of age, adolescent girls,
pregnant women and lactating mothers.

Bacteria with antimicrobial properties

Scientists in Kerala have completed the whole genome sequencing of a rare


bacterium capable of producing antifungal and insecticidal compounds.

Biocontrol

Biological control​ or ​biocontrol​ is a method of ​controlling pests​ such as ​insects​,


mites​, ​weeds​ and ​plant diseases​ ​using other organisms​.[1]​
​ It relies on ​predation​,
parasitism​, ​herbivory​, or other natural mechanisms, but typically also involves an active
human management role. It can be an important component of ​integrated pest
management​ (IPM) programs.

“Reducing Food Loss and Waste” Report

● It is a new report published by the World Resources Institute (WRI) with the
support of the Rockefeller Foundation.
● It has quantified global food wastage.

© IAS.NETWORK
7
● It put forward a Global Action Agenda that calls on governments, companies,
farmers and consumers to collectively overcome “the world’s food loss and waste
problem.”
● Some of these actions include developing national strategies for food loss and
waste reduction, creating national PPP, launch supply chain initiatives, reducing
small-holder losses and shifting consumer social norms.

Globally uneaten food

● Nearly one-third of the food that is produced each year goes uneaten, costing the
global economy over $940 billion.
● The uneaten food is responsible for emitting about 8 per cent of planet-warming
greenhouse gases into the atmosphere, said the report.
● Referencing “numerous studies”, the report said most of the food loss happens
“near the farm” predominantly in lower-income countries.
● And most of the food waste happens “near the plate” predominantly in
higher-income countries.

Most perishable items

● Fruits and vegetables follow, with over 41%.


● When viewed as a proportion, by weight, of all the food estimated to be lost and
wasted globally, fruits and vegetables make up the largest share of total annual
food loss and waste.
● Using data from the Food and Agriculture Organization of the UN, the report
concluded that roots and tubers are the food group that face the maximum
wastage, at over 62% for 2007.

AUMX: 1st ASEAN-US Maritime Exercise

© IAS.NETWORK
8
The first ASEAN-US Maritime Exercise (AUMX) between regional bloc- Association of
Southeast Asian Nations (ASEAN) and United States kicked off at the Sattahip Naval
Base in Thailand.

Project REPLAN
Project REPLAN (REducing PLAstic in Nature)

● KVIC, as part of its commitment to Swachh Bharat Abhiyaan, had started


manufacturing of plastic-mixed handmade paper under its project REPLAN
(REducing PLAstic in Nature).
● In this project, the waste plastic is collected, cleaned, chopped, beaten and
treated for softness.
● After that, it is mixed with the paper raw material i.e. cotton rags pulp in a ratio of
80 % (pulp) and 20% (plastic waste).
● The institute has sold over six lakh handmade plastic mixed carry bags since
September 2018

Nuakhai Juhar

Nuakhai Juhar, a harvest festival welcoming the newly harvest crop of the season was
celebrated in Odisha. It is one of the most ancient festivals celebrated in Odisha to
welcome new crop of the season. It is an important social festival of Western Odisha
and is an agricultural festival

ANDREX Project

The project seeks ​to assess the role of the Weddell gyre​ in driving the southern closure
of the meridional overturning circulation, in ventilating the deep global ocean, and in
sequestering carbon and nutrients in the global ocean abyss​.

© IAS.NETWORK
9
Asteroid Impact Deflection Assessment

● Asteroid Impact Deflection Assessment (AIDA) is a joint research mission


between NASA and the European Space Agency (ESA) teams.
● It aims to study the viability of diverting an asteroid by crashing a spacecraft into
its surface.
● The project aims to deflect the orbit of one of the two Didymos asteroids between
Earth and Mars, with an observer craft gauging the effect of the impact more
effectively than ground-based observers could manage.
● The target is the smaller of two bodies in the “double Didymos asteroids” that are
in orbit between Earth and Mars.
● Didymos is a near-Earth asteroid system. Its main body measures about 780 m
across; the smaller body is a “moonlet” about 160 m in diameter.
● The project aims to deflect the orbit of the smaller body through an impact by one
spacecraft.
● Then a second spacecraft will survey the crash site and gather the maximum
possible data on the effect of this collision.

Ethanol Blended Petrol (EBP) Programme

The Union Cabinet has approved a higher procurement price for ethanol purchased by
oil marketing companies for the ethanol blended petrol (EBP) programme

● According to the new decision, the price of ethanol from C-heavy molasses will
be increased from ₹43.46 per litre to ₹43.75 per litre.
● The price of ethanol from B-heavy molasses will be increased from ₹52.43 per
litre to ₹54.27 per litre. The price of ethanol from sugarcane juice, sugar, and
sugar syrup has been set at ₹59.48 per litre.
● The oil marketing companies will also pay the GST and transportation tax
associated with the ethanol supply — a provision that existed in the previous plan
as well.

EBP Programme

© IAS.NETWORK
10
● Ethanol Blended Petrol (EBP) programme was launched in January, 2003 for
supply of 5% ethanol blended Petrol.
● The programme sought to promote the use of alternative and environment
friendly fuels and to reduce import dependency for energy requirements.
● OMCs are advised to continue according priority of ethanol from 1) sugarcane
juice/sugar/sugar syrup, 2) B-heavy molasses 3) C-heavy molasses and 4)
damaged food grains/other sources.
● At present, this programme has been extended to whole of India except UTs of
Andaman Nicobar and Lakshadweep islands with effect from 01st April, 2019
wherein OMCs sell petrol blended with ethanol up to 10%.

Ethanol

● Ethanol, an anhydrous ethyl alcohol having chemical formula of C​2​H​5​OH, can be


produced from sugarcane, maize, wheat, etc which are having high starch
content.
● In India, ethanol is mainly produced from sugarcane molasses by fermentation
process.
● Ethanol can be mixed with gasoline to form different blends.
● As the ethanol molecule contains oxygen, it allows the engine to more completely
combust the fuel, resulting in fewer emissions and thereby reducing the
occurrence of environmental pollution.

Single-Use Plastic

● On this Independence Day address, PM called for a movement to eliminate


single-use plastic in India, beginning on Gandhi Jayanti (October 2).
● The move is part of an ambitious drive against Single-Use Plastic (SUP), under
the theme “Shramdaan”, for which a detailed plan has been worked out for
ministries and departments.

Single-use plastic

© IAS.NETWORK
11
● As the name suggests, single-use plastics (SUPs) are those that are discarded
after one-time use.
● Besides the ubiquitous plastic bags, SUPs include water and flavoured/aerated
drinks bottles, takeaway food containers, disposable cutlery, straws, and stirrers,
processed food packets and wrappers, cotton bud sticks, etc.
● Of these, foamed products such as cutlery, plates, and cups are considered the
most lethal to the environment.

Plastic waste management

● The Plastic Waste Management Rules, 2016 notified by the Centre called for a
ban on “non-recyclable and multi-layered” packaging by March 2018, and a ban
on carry bags of thickness less than 50 microns.
● The Rules were amended in 2018, with changes that activists say favoured the
plastic industry and allowed manufacturers an escape route. The 2016 Rules did
not mention SUPs.
● On World Environment Day in 2018, India pledged to phase out SUPs by 2022.
● The PM has called for “a new revolution against plastic”, and some
government-controlled bodies such as Air India and the Indian Railways have
announced they would stop SUPs.

Global Liveability Ranking 2019

● New Delhi has dropped by six places to rank 118th on a list of the world’s most
liveable cities due to increase in cases of petty crimes and poor air quality.
● While New Delhi registered the biggest decline in Asia, Mumbai also fell two
places since last year to rank 119th on the list topped by Vienna (Austria) for the
second consecutive year.
● The list is topped by Vienna (Austria) for the second consecutive year.

About the ranking

● The Economist Intelligence Unit (EIU) publishes an annual Global Liveability


Ranking.

© IAS.NETWORK
12
● The EIU ranking of 140 cities is based on their scores in five broad categories —
stability, healthcare, culture and environment, education, and infrastructure.
● Each factor in a city is rated as acceptable, tolerable, uncomfortable, undesirable
or intolerable.

The survey rates cities worldwide based on 30 qualitative and quantitative criteria,
which fall into five general categories:

1. Stability
2. Health care.
3. Culture and environment.
4. Education
5. Infrastructure

Reasons for decline in liveability in Indian cities:

1. Abuses against journalists.


2. Rise in Crime rates.
3. Climatic changes.
4. Constrained liveability conditions

Government to develop a master plan for Tigers at High altitude


● Union Environment Ministry released a report on Status of Tiger Habitats in high
altitude ecosystems.

About the study

● The study is led by the Global Tiger Forum (GTF), with range country
governments of Bhutan, India and Nepal, along with WWF.
● It has been supported by the Integrated Tiger Habitat Conservation Programme
(ITHPC) of the IUCN.
● This provides the action strategy for a high altitude tiger master plan, with gainful
portfolio for local communities.

© IAS.NETWORK
13
● It ensures centrality of tiger conservation in development, through an effective
coordination mechanism, involving stakeholders and line departments operating
within the landscape.

EX TSENTR 2019

Armies of India, Pakistan will take part in this year’s military exercise TSENTR 2019 that
will be conducted by Russia from September 9 to 23, 2019. This will be the first time
that forces from India and Pakistan will participate in multilateral exercise together since
airstrikes conducted by Indian force, deep in Pakistan’s Balakot district in February
2019. Earlier armies of India and Pakistan had met last year as part of military exercise
conducted by Shanghai Cooperation Organisation (SCO) – also held in Russia.

SATHI: IIT Kharagpur to assist academic institutes in neighbouring region

Sophisticated Analytical and Technical Help Institute (SATHI)

SATHI is a state-of-the-art, professionally managed science and technology


infrastructure facility. Its primary objective will be to extend help to academic institutes in
neighbouring regions.

As per Department of Science and Technology mandate, the SATHI Centre will run
24×7 round the year. At least 70% of instrument use time will be reserved for external
users from other start-ups, entrepreneurs, academic institutes, industry and national
laboratories.

Facility would involve highest level of expertise of IIT KGP in several key convergence
domains of science and technology like structural and safety engineering, medical
sciences, soft materials, advanced communication, nano technology and quantum
photonics.

© IAS.NETWORK
14
Mid- Monsoon 2019 Lightning Report

To map lightning across India and ​the lives they have claimed.

It has been prepared by Climate Resilient Observing Systems Promotion Council


(CROPC), a non-profit organisation that works closely with India Meteorological
Department (IMD)

What has the report found?

● Lightning strikes have caused at least 1,311 deaths in the four-month period
between April and July this year, according to a first-of-its-kind report on lightning
incidents in India.
● UP accounted for 224 of these deaths, followed by Bihar (170), Odisha (129) and
Jharkhand (118).
● It counted 65.55 lakh lightning strikes in India during this four-month period, of
which 23.53 lakh (36 per cent) happened to be cloud-to-ground lightning, the kind
that reaches the Earth.
● The other 41.04 lakh (64 per cent) were in-cloud lightning, which remains
confined to the clouds in which it was formed.

Centralised Technology Vertical (CTV)

The centre is planning to set up a Centralised Technology Vertical (CTV) under the
Central Bureau of Investigation (CBI)

© IAS.NETWORK
15
-It facilitates real-time information sharing with investigators across the country.

Eat Right India movement

It was launched by the Food Safety and Standards Authority of India (FSSAI).

The movement aims to cut down salt/sugar and oil consumption by 30% in three
years.It also aims to engage and enable citizens to improve their health and well-being
by making the right food choices.

The Eat Right India movement acts as a crucial preventive healthcare measure to
trigger social and behavioural change through a judicious mix of regulatory measures,
combined with soft interventions for ensuring awareness and capacity building of food
businesses and citizens alike.

Eastern Economic Forum

India extends $1 billion line of credit for development of Russia’s Far East and was
announced at the ongoing ​5th Eastern Economic Forum​.

About the Eastern Economic Forum:

1. Established by Decree of the President of the Russian Federation in 2015.


2. It takes place each year in Vladivostok.
3. Serves as a platform for the discussion of key issues in the world economy,
regional integration, and the development of new industrial and technological
sectors, as well as of the global challenges facing Russia and other nations.
4. Participants: The Forum business programme includes a number of business
dialogues with leading partner countries in the Asia-Pacific region, and with
ASEAN, a key integration organization of dynamically developing nations in
Southeast Asia.

© IAS.NETWORK
16
ABOUT THE FAR EAST:

The Far East is the easternmost part of Russia.

It borders two oceans, the Pacific and the Arctic, and five countries (China, Japan,
Mongolia, the United States and the DPRK).

The Far Eastern Federal District covers more than a third of the country’s territory.

RESOURCES: The Far East is rich in natural resources like diamonds, stannary, borax
materials, 50 gold, tungsten, and fish and seafood. About 1/3 of all coal reserves and
hydro-engineering resources of the country are here. Forests of the region comprise
about 30% of the total forest area of Russia.

© IAS.NETWORK
17
Hepatitis B

Bangladesh, Bhutan, Nepal and Thailand became the first four countries in the World
Health Organization’s southeast Asia region to have successfully controlled hepatitis B.

Hepatitis means inflammation of the liver. When the liver is inflamed or damaged, its
function can be affected.

Causes: Heavy alcohol use, toxins, some medications, and certain medical conditions
can all cause hepatitis. However, hepatitis is often caused by a virus. In the United
States, the most common hepatitis viruses are hepatitis A virus, hepatitis B virus, and
hepatitis C virus.

What is the difference between hepatitis A, hepatitis B, and hepatitis C?

Hepatitis A, Hepatitis B, and Hepatitis C are liver infections caused by three different
viruses.

1. Hepatitis A is usually a short-term infection and does not become a long-term


infection.
2. Hepatitis B and hepatitis C can also begin as short-term infections but in some
people, the virus remains in the body, and causes chronic, or lifelong, infection.
3. There are vaccines to prevent hepatitis A and hepatitis B; however, there is no
vaccine for hepatitis C.

Germany to phase out glyphosate by 2023

Germany took this step to protect insect populations that play a pivotal role in
ecosystems and pollination of food crops.Glyphosate is used widely in agriculture,
forestry, urban and home applications.

© IAS.NETWORK
18
Concerns:​ Its use has been surrounded by much debate due to safety concerns. The
World Health Organization’s International Agency for Research on Cancer (IARC) had
declared glyphosate it “probably” carcinogenic to humans.

Travel and Tourism Competitive Index 2019


-released by the World Economic Forum (WEF) ranking the travel & tourism
competitiveness of 140 economies was recently released.
-The study scores countries on four indicators —​ enabling environment; travel and
tourism policy and enabling conditions; infrastructure; natural and cultural rankings.

National Animal Disease Control Programme

Government has launched National Animal Disease Control Programme for Foot and
Mouth Disease and Brucellosis
● The programme for Foot and Mouth Disease and Brucellosis is a 100% centrally
funded programme, with a total outlay of Rs.12,652 crore from 2019 to 2024.
● It aims to control Foot and Mouth Disease and Brucellosis by 2025 with
vaccination and eventual eradication by 2030.

About FMDs

● Foot and mouth disease (FMD) is a severe, highly contagious viral disease of
livestock that has a significant economic impact.
● The disease affects cattle, swine, sheep, goats and other cloven-hoofed
ruminants.
● Intensively reared animals are more susceptible to the disease than traditional
breeds.
● The disease is rarely fatal in adult animals, but there is often high mortality in
young animals due to myocarditis or, when the dam is infected by the disease,
lack of milk.
● FMD is characterised by fever and blister-like sores on the tongue and lips, in the
mouth, on the teats and between the hooves.
● The disease causes severe production losses, and while the majority of affected
animals recover, the disease often leaves them weakened and debilitated.

© IAS.NETWORK
19
● FMD is found in all excretions and secretions from infected animals. Notably,
these animals breathe out a large amount of aerosolised virus, which can infect
other animals via the respiratory or oral routes.
● The virus may be present in milk and semen for up to 4 days before the animal
shows clinical signs of disease

Indian Ocean Conference

Fourth Indian Ocean Conference 2019 was recently held in Male, capital of Maldives.

About Indian Ocean Conference:

Initiated by India Foundation along with its partners from Singapore, Sri Lanka and
Bangladesh.It is an annual effort to bring together Heads of States/Governments,
Ministers, Thought Leaders, scholars, diplomats, bureaucrats and practitioners from
across the region.

Great Indian Bustards (GIB)

SC directed government to prepare a plan for the protection of the Great Indian Bustard:
NGT.

Key facts:

1. IUCN status: critically endangered.


2. Found in Gujarat, Maharashtra, Karnataka and Andhra Pradesh.
3. Listed in Schedule I of the Indian Wildlife (Protection)Act, 1972 and in the CMS
Convention and in Appendix I of CITES.
4. Identified as one of the species for the recovery programme under the Integrated
Development of Wildlife Habitatsof the Ministry of Environment and Forests.
5. Project Great Indian Bustard — state of Rajasthan — identifying and fencing off
bustard breeding grounds in existing protected areas as well as provide secure
breeding enclosures in areas outside protected areas.

© IAS.NETWORK
20
6. Protected areas:Desert National Park Sanctuary — Rajasthan, Rollapadu Wildlife
Sanctuary – Andhra Pradesh and Karera Wildlife Sanctuary– Madhya Pradesh.

Nilgiri Tahr
In more good news for the State animal, the Nilgiri tahr, its sightings in the Mukurthi
National Park have risen from 568 in 2018 to 612 this year.

● There was a decrease in tahr numbers in 2017, when a population of only 438
was recorded, down from 480 in 2016.
● This was the second consecutive year that an increase in the population of the
animal had been recorded in the park, meaning the population of the Nilgiri tahr,
also known as the Nilgiri ibex, has risen by 132 since 2016.

Nilgiri tahr

● IUCN Conservation Status: Endangered


● The Nilgiri tahr (Nilgiritragus hylocrius) aka the Nilgiri ibex or simply ibex.
● It is endemic to the Nilgiri Hills and the southern portion of the Western Ghats in
the states of Tamil Nadu and Kerala in Southern India.
● It is the state animal of Tamil Nadu.
● The Nilgiri tahr inhabits the open montane grassland habitat of the South
Western Ghats montane rain forests eco-region.
● At elevations from 1,200 to 2,600 metres (3,900 to 8,500 ft), the forests open into
grasslands interspersed with pockets of stunted forests, locally known as sholas.
● Eravikulam National Park is home to the largest population of this Tahr.

National Genomic Grid (NGG)

1. It will study genomic data of cancer patients from India.


2. It will collect samples from cancer patients, through a network of pan-India
collection centres by bringing all cancer treatment institutions on board.

© IAS.NETWORK
21
3. The grid to be formed will be in line with the National Cancer Tissue Biobank
(NCTB) set up at the Indian Indian Institute of Technology, Madras.

NCTB

National Cancer Tissue Biobank (NCTB), is a joint initiative of the Department of


Science and Technology (DST), Government of India and Indian Institute of
Technology, Madras.

● It collects cancer tissue samples with consent from patients diagnosed with
cancer.
● The aim is to provide researchers with high quality of cancer tissues and the
patient data in order to facilitate cancer research that will lead to improvements in
cancer diagnosis and treatment.
● This research is carried out through the technique of Genome Sequencing.

3 Animal Species In India Extinct Due To Desertification:

Three species of animals — the Indian Cheetah, pink-headed duck, and the Great
Indian Bustard — have become extinct.

Time Bank

Madhya Pradesh government’s Happiness Department plans to set up a Time Bank.

It would lend currency to an hour, which could be exchanged to learn a new skill without
the need for any paper money.

What is a time bank? It is a reciprocity-based work trading system in which hours are
the currency. With time banking, a person with one skill set can bank and trade hours of

© IAS.NETWORK
22
work for equal hours of work in another skill set instead of paying or being paid for
services.

How it operates? Whenever a bank member needs a service or wants to acquire a skill,
say gardening or playing a guitar, she could exchange a credit, worth an hour, with
another member knowing the skill.

First Time Bank was set up in Japan in 1973. Today, there are more than 500 such
communities across 32 countries.

‘SLINEX 2019’

The ‘SLINEX 2019’, the bilateral maritime exercise between India and Sri Lanka

Task Force for drawing up National Infrastructure Pipeline

Central government constituted a task force for drawing up a roadmap for ‘National
Infrastructure Pipeline’ (NIP) of Rs.100 lakh crore from financial year 2019-20 to
2024-25. The Task Force is headed by Economic Affairs Secretary Atanu Chakraborty
would cover greenfield and brownfield projects costing above `Rs.100 crore each.

About Task Force Objective:

To identify technically feasible and economically viable infrastructure projects that can
be initiated during current fiscal year. Task force would draw up a list of infrastructure
projects that can be set rolling in 2019-20 as it looks at steps to tide over the economic
slowdown.Each Ministry/ Department would be responsible for the monitoring of
projects so as to ensure their timely and within-cost implementation.

It will help in stepping-up annual infrastructure investment to achieve the Gross


Domestic Product (GDP) of $5 trillion by 2024-25.

Hurricane Dorian

© IAS.NETWORK
23
India has announced an immediate humanitarian assistance of $1 million to help people
in ​The Bahamas

PM raises India’s target to restore degraded land to 26 million hectares by


2030

The announcement was made by PM Modi while speaking at the high level segment at
14th session of Conference of Parties (COP) to United Nations Convention to Combat
Desertification (UNCCD). UNCCD COP-14 is being hosted by India this time and being
held at Greater Noida.

Agenda of UN conference is to reverse degradation of land and fix critical gaps in its
management.

India will also setup a global technical support institute for UNCCD member countries
for their capacity building and support regarding Land Degradation Neutrality.

Pradhan Mantri Kisan Maan Dhan Yojana

Features of the scheme:

1. The scheme is voluntary and contributory for farmers in the entry age group of 18
to 40 years.
2. A monthly pension of Rs. 3000/– will be provided to them on attaining the age of
60 years. The spouse is also eligible to get a separate pension of Rs.3000/- upon
making separate contributions to the Fund.
3. Initial contribution: The farmers will have to make a monthly contribution of Rs.55
to Rs.200, depending on their age of entry, in the Pension Fund till they reach the
retirement date i.e. the age of 60 years.
4. Farmer’s monthly contribution can be made from instalments of PM-KISAN
(Pradhan Mantri Kisan Samman Nidhi Yojana) or through Common Service
Centres (CSCs)

© IAS.NETWORK
24
5. The Central Government will also make an equal contribution of the same
amount in the pension fund.
6. The Life Insurance Corporation of India (LIC) shall be the Pension Fund
Managerand responsible for Pension pay out.
7. If there is no spouse, then total contribution along with interest will be paid to the
nominee.
8. If the farmer dies after the retirement date, the spouse will receive 50% of the
pension as Family Pension.
9. After the death of both the farmer and the spouse, the accumulated corpus shall
be credited back to the Pension Fund.
10. The beneficiaries may opt voluntarily to exit the Scheme after a minimum period
of 5 years of regular contributions.
11. In case of default in making regular contributions, the beneficiaries are allowed to
regularize the contributions by paying the outstanding dues along with prescribed
interest

UN​ Peacekeeping
India calls for reforms in peacekeeping at UNSC debate

What is peacekeeping?

United Nations Peacekeeping was created in 1948. Its first mission involved the
establishment of the UN Truce Supervision Organization (UNTSO), which served to
observe and maintain ceasefire during the 1948 Arab-Israeli War.

UN peacekeeping is a unique global partnership. It brings together the General


Assembly, the Security Council, the Secretariat, troop and police contributors and the
host governments in a combined effort to maintain international peace and security. Its
strength lies in the legitimacy of the UN Charter and in the wide range of contributing
countries that participate and provide precious resources.

© IAS.NETWORK
25
-​Peacekeeping by the United Nations is a unique and dynamic instrument developed by
the organization as a way to help countries torn by conflict to create the conditions for
lasting peace.

-It is distinguished from peacebuilding, peacemaking, and peace enforcement although


the UN does acknowledge that all activities are “mutually reinforcing” and that overlap
between them is frequent in practice.

-Peacekeepers monitor and observe peace processes in post-conflict areas and assist
ex-combatants in implementing the peace agreements they may have signed.

-UN peacekeepers often referred to as Blue Berets or Blue Helmets because of their
light blue berets or helmets can include soldiers, police officers, and civilian personnel.

India Iodine Survey 2018-19 Report


The survey was conducted by Nutrition International in collaboration with the AIIMS and
the Indian Coalition for the Control of Iodine Deficiency Disorders (ICCIDD).

Key findings:

1. Gujarat produces 71% of salt in the country, followed by Rajasthan at 17% and
Tamil Nadu at 11%.
2. 76.3% of Indian households consumed adequately iodised salt.
3. Tamil Nadu (61.9%) has the lowest consumption of iodized salt despite being the
third biggest producer of salt in the country.
4. It is followed by Andhra Pradesh (63.9%), Rajasthan (65.5%), Odisha (65.8%)
and Jharkhand (68.8%).
5. Only 13 out of 36 States have achieved Universal Salt Iodisation or have 90% of
households with access to adequately iodised salt.

© IAS.NETWORK
26
‘ANGAN’- International Conference on Energy Efficiency in Building Sector

The recently held international conference, ANGAN (Augmenting Nature by Green


Affordable New-habitat), focussed on Energy Efficiency in Building Sector.

● Organised by the Bureau of Energy Efficiency (BEE), Ministry of Power,


Government of India in collaboration with GIZ under the Indo German Technical
Cooperation.
● It’s aim is to provide a platform to deliberate on interdependence between
organizations, systemic sustainability and feedback loops for better resource
efficiency.

African Swine Fever (ASF)

ASF outbreak ​has been sweeping through swine populations in China .

1. ASF is a highly contagious and fatal animal disease that infects domestic and
wild pigs, typically resulting in an acute form of hemorrhagic fever.
2. It was first detected in Africa in the 1920s.
3. The mortality is close to 100 per cent, and since the fever has no cure, the only
way to stop it spreading is by culling the animals.
4. ASF is not a threat to human beings since it only spreads from animals to other
animals.

Market Intervention Price Scheme

1. I​t is a price support mechanism implemented on the request of State


Governments.
2. It is for procurement of perishable and horticultural commodities in the event of a
fall in market prices.
3. The Scheme is implemented when there is at least 10% increase in production or
10% decrease in the ruling rates over the previous normal year.
4. Its objective is to protect the growers of these horticultural/agricultural
commodities from making distress sale in the event of bumper crop during the
peak arrival period when prices fall to very low level.

© IAS.NETWORK
27
5. The Department of Agriculture & Cooperation is implementing the scheme.

Funding

-Under MIP, funds are not allocated to the States.

-Instead, central share of losses as per the guidelines of MIP is released to the State
Governments/UTs, for which MIP has been approved, based on specific proposals
received from them.

The area of operation is restricted to the concerned state only.

The MIS has been implemented in case of commodities like apples, kinnoo/malta,
garlic, oranges, galgal, grapes, mushrooms, clove, black pepper, pineapple, ginger,
red-chillies, coriander seed etc.

IPCC report on the Ocean and Cryosphere in a Changing Climate


It underlined the dire changes taking place in oceans, glaciers and ice-deposits on land
and sea.

Findings

● Over the 21st century, the ocean is projected to transition to unprecedented


conditions with increased temperatures, further ocean acidification, marine
heatwaves and more frequent extreme El Niño and La Niña events,” according to
the report.
● It is virtually certain that the global ocean has warmed unabated since 1970 and
has taken up more than 90% of the excess heat in the climate system (high
confidence).
● Since 1993, the rate of ocean warming has more than doubled.
● Marine heatwaves have very likely doubled in frequency since 1982 and are
increasing in intensity, the report notes.

© IAS.NETWORK
28
● The Southern Ocean accounted for 35%–43% of the total heat gain in the upper
2,000 m global ocean between 1970 and 2017, and its share increased to
45%–62% between 2005 and 2017.
● Globally sea levels are estimated to rise 1.1 metre by 2100, if countries are not
able to restrict emissions “well below” 2 degrees Celsius above pre-industrial
levels.

Quantum Supremacy
● It refers to a quantum computer solving a problem that cannot be expected of a
classical computer in a normal lifetime.
● This relates to the speed at which a quantum computer performs.
● The phrase “quantum supremacy” was coined in 2011 by John Preskill,
Professor of Theoretical Physics at the California Institute of Technology in a
speech.
● According to reports the quantum processor took 200 seconds to perform a
calculation that the world’s fastest supercomputer, Summit, would have taken
10,000 years to accomplish.

What is quantum computing?

● Quantum computing takes advantage of the strange ability of subatomic particles


to exist in more than one state at any time.
● Due to the way the tiniest of particles behave, operations can be done much
more quickly and use less energy than classical computers.

How is Quantum computer different from a traditional computer?

● What differentiates a quantum computer from a traditional computer is the way


the two store information.
● Quantum computers perform calculations based on the probability of an object’s
state before it is measured – instead of just 1s or 0s – which means they have
the potential to process exponentially more data compared to classical
computers.

© IAS.NETWORK
29
● Classical computers carry out logical operations using the definite position of a
physical state.
● These are usually binary, meaning its operations are based on one of two
positions. A single state – such as on or off, up or down, 1 or 0 – is called a bit.
● In quantum computing, operations instead use the quantum state of an object to
produce what’s known as a qubit.
● These states are the undefined properties of an object before they’ve been
detected, such as the spin of an electron or the polarisation of a photon.

PM receives ‘Global Goal Keeper Award’ for Swachh Bharat Abhiyan

About the award

● ‘Goalkeepers’ is an initiative of the Bill and Melinda Gates Foundation.


● Its aim is to bring together leaders from around the world to accelerate progress
toward achieving the Sustainable Development Goals (SDG).
● The organization also provides reports and data flow charts over SDGs progress.

Exercise Malabar

● Exercise Malabar is a trilateral naval exercise involving the United States, Japan
and India as permanent partners.
● Originally begun in 1992 as a bilateral exercise between India and the United
States, Japan became a permanent partner in 2015.

Cryodrakon Boreas

Paleontologists have identified a new species, named it Cryodrakon boreas, and


declared that ​it could be one of the largest flying animals​.

© IAS.NETWORK
30
With a wingspan of over 10 metres, it is believed to have flown over the heads of
dinosaurs.The reptile lived over 77 million years ago in what is western ​Canada​ today.

Basel Ban Amendment

The ​1995 Basel Ban Amendment​, a global waste dumping prohibition, has become an
international law after ​Croatia (​97​th​ country to ratify)​ ratified it on September 6, 2019​.

Basel Convention ​— Control of Transboundary Movements of Hazardous Wastes and


Their Disposal:

● Opened for signature on 22 March 1989


● entered into force on 5 May 1992
● Parties — 187.
● It is an international treaty that was designed to reduce the movements of
hazardous waste between nations, and specifically to prevent transfer of
hazardous waste from developed to less developed countries (LDCs).
● It does not address the movement of radioactive waste.

Snow leopard

● Listed as Vulnerable on the IUCN Red List of Threatened Species.


● Inhabit alpine and subalpine zones at elevations from 3,000 to 4,500 m (9,800 to
14,800 ft).
● State animal of Himachal Pradesh and the National Heritage Animal of Pakistan.
● Habitat extends through twelve countries: Afghanistan, Bhutan, China, India,
Kazakhstan, Kyrgyzstan, Mongolia, Nepal, Pakistan, Russia, Tajikistan, and
Uzbekistan. China contains as much as 60% of all snow leopard habitat areas.
● Llisted on Appendix I of the Convention on International Trade of Endangered
Species (CITES).
● Global Snow Leopard Forum, 2013:12 countries encompassing the snow
leopard’s range (Afghanistan, Bhutan, China, India, Kazakhstan, Kyrgyz
Republic, Mongolia, Nepal, Pakistan, Russia, Tajikistan, and Uzbekistan).
● Bishkek Declaration:To protect the species and it’s environment.

© IAS.NETWORK
31
● Global Snow Leopard and Eco-system Protection Program:It is a joint initiative of
range country governments, international agencies, civil society, and the private
sector. Goal — secure the long-term survival of the snow leopard in its natural
ecosystem.

Fall Armyworm (FAW)

Maize crops falling victim to fall armyworm in Bihar. Reports of the pest attacking crops
have been reported from a number of districts in the state, India’s third-largest maize
producer.

FAW

It is a native of the tropical and sub-tropical regions of the Americas.

First detected in the African continent in 2016. Since then, it has spread to other
countries such as China, Thailand, Malaysia and Sri Lanka.

In India: It was reported in India for the first-time in Karnataka. Within a span of only six
months, almost 50 per cent of the country, including Mizoram, Maharashtra, Karnataka,
Tamil Nadu, Andhra Pradesh, Chhattisgarh, Madhya Pradesh, Gujarat and West
Bengal, has reported FAW infestations.

UNCCD COP14: Drought Toolbox launched

About Drought Toolbox

● It is a one-stop shop for all actions on drought


● It is a kind of knowledge bank that may be used by vulnerable countries, such as India, to
reduce drought risk and be better prepared and effectively respond to it.

© IAS.NETWORK
32
● It is a web page that provides involved stakeholders easy access to case studies and other
resources to support action on drought preparedness with aim to boost resilience of
people and ecosystems to drought.
● It contains tools that strengthen ability of vulnerable countries and enable communities to
anticipate and prepare for drought effectively, mitigate their impacts and find land
management tools that help them to build resilience to drought.
● It will help countries in framing/fine-tuning their respective national drought policies in
due course based on monitoring, forecast, and early warning​.

Maitree 2019

Indo-Thailand Joint Military Exercise to be held in Meghalaya

Curriculum for Life Skills (Jeevan Kaushal)

Designed by University Grants Commission (UGC).

Objectives​: The curriculum is designed to impart and strengthen the knowledge, skills
and dispositions believed to be the best requisites of the current Industry and thereby
empower the talent inherent in each learner.

Life skills (JeevanKaushal) curriculum cover the set of human talents acquired by an
individual via classroom learning or life experience that can help them to deal with
problems encountered in day to day life. This includes the core skills each individual
must possess internally as well as externally for the betterment of self and the society
as a whole. Adoption of life skills is the key to success and quality in life.

K2-18b
K2-18b is now the only planet orbiting a star outside the Solar System known to have
both water and temperatures that could be potentially habitable​.

© IAS.NETWORK
33
Not ‘Earth 2.0’

● K2-18b is not ‘Earth 2.0’ as it is significantly heavier and has a different


atmospheric composition.
● For one thing, K2-18b’s size and surface gravity are much larger than Earth’s. Its
radiation environment may be hostile.

World University Rankings 2019


● The World University Rankings was recently released by the UK-based Times
Higher Education.
● Oxford University continues to lead the rankings table followed by California
Institute of Technology and University of Cambridge. Stanford University and MIT
complete the top five table.

No Indian university this year

● For the first time since 2012, no Indian institution featured among the top 300.
● The country’s best performing institution, IISc-Bangalore, slipped 50 places from
the 251-300 ranking cohort into the 301-350 bracket.
● The dip was on account of a significant fall in its citation impact score offsetting
improvements in research environment, teaching environment and industry
income.

National Pension Scheme for Traders and Self Employed Persons

Ministry of Labour & Employment has launched ​National Pension Scheme for
Traders and Self Employed Persons, a pension scheme for the Vyaparis
(shopkeepers/retail traders and self-employed persons),

About the Scheme

© IAS.NETWORK
34
● It is a voluntary and contributory pension scheme for entry age of 18 to 40 years
with a provision for minimum assured pension of Rs 3,000/- monthly on attaining
the age of 60 years.
● The eligible Vyaparis can visit their nearest CSCs and get enrolled under the
scheme. In addition people can also self-enroll by visiting its portal.
● At the time of enrollment, the beneficiary is required to have an Aadhaar card
and a saving bank/ Jan-dhan Account passbook only.
● He/ She should be within 18 to 40 years of age group. GSTIN is required only for
those with turnover above Rs. 40 lakhs.
● The enrolment under the scheme is free of cost for the beneficiaries. The
enrolment is based upon self-certification.
● An estimated 3 crore Vyaparis in the country are expected to be benefitted under
the pension scheme.

Eligibility Criteria

● Vyaparis with annual turnover not exceeding Rs 1.5 crore are eligible for the
pension.
● The beneficiary should not be income tax payer and also not a member of
EPFO/ESIC/NPS (Govt.)/PM-SYM.
● The Central Government shall give 50 % share of the monthly contribution and
remaining 50% contribution shall be made by the beneficiary.
● The monthly contribution is kept low to make it affordable. For example, a
beneficiary is required to contribute as little as Rs.100/- per month at a median
entry age of 29 years.

Amendments to ​District Mineral Foundation (DMF) Trust Rules, 2015

Amendments to District Mineral Foundation (DMF) Trust Rules, 2015, by Chhattisgarh


government has made it more inclusive, people-centric and will also empower people
affected by mining in the state,

© IAS.NETWORK
35
Chhattisgarh became the ​first state in July 2019, to amend DMF rules​.

● The new rule mandates the ​inclusion of 10 Gram Sabha members directly
from mining-affected areas​ in the DMF Governing Council (GC).
● In Scheduled Areas, at least 50 per cent of the Gram Sabha members must be
from Scheduled Tribes (ST).
● To ensure better public accountability, a two-step social audit process has been
mandated.
● Provisions have also been introduced for five-year plan, which can be subjected
to a third party review if the secretary of the mines department considers it to be
necessary.
● The rules have also specified ‘​sustainable livelihood​’ as a high priority issue,
including for forest rights holders.

About DMFs:

● DMFs were ​instituted under the Mines and Minerals (Development and
Regulation) (MMDR) Amendment Act 2015​.
● They are ​non-profit trusts ​to work for the interest and benefit of persons and
areas affected by mining-related operations in such manner as may be
prescribed by the State Government.
● Jurisdiction​: Its manner of operation comes under the jurisdiction of the relevant
State Government.

The various state DMF rules and the Pradhan Mantri Khanij Khestra Kalyan
Yojana (PMKKKY) guidelines stipulate some “high priority” issues for DMFs,
including:

1. Drinking water.
2. Health
3. Women and child welfare.
4. Education
5. Livelihood and skill development.

© IAS.NETWORK
36
6. Welfare of aged and disabled.
7. Sanitation

Salmonella

A group of bacteria that can cause food-borne illnesses known as salmonellosis.The


World Health Organisation (WHO) identifies Salmonella as one of four key global
causes of diarrhoeal diseases.

Jan Soochna Portal-2019

It is the first-ever public information portal launched in Rajasthan.

Aim​: To provide information about government authorities and departments suo motu to
the public in the true spirit of the Right To Information Act. It is ​a portal for quick access
to information on government departments.

Bamboonomics

It is a movement to combat desertification and climate change which will involve the
tribal community of India since they have the expertise in this field.

The movement will ensure that tribals can earn a livelihood without causing
environmental harm.It was launched at the COP 14 of the United Nations Convention to
Combat Desertification (UNCCD).

Delhi Declaration to restore degraded land by 2030


Delhi Declaration

● The Delhi Declaration, a consensus document, agreed upon by more than 100
countries “welcomed” the proposed adoption of a “voluntary” land degradation
neutrality target by India.
● India has committed to restoring at least 26 million hectares of degraded land by
2030. The Declaration doesn’t detail commitments by other countries.

© IAS.NETWORK
37
● Almost 122 nations, including India, have made voluntary commitments in
previous years to ensure that a certain percentage of their degraded land was
restored.
● India had agreed, again on a voluntary basis, to restore 20 million hectares by
2020.
● Nearly 96 million hectares of land is deemed ‘degraded’ in India.
● Countries will address insecurity of land tenure, promote land restoration to
reduce land-related carbon emissions and mobilise innovative sources of finance
from public and private sources.

Him Vijay Exercise

Indian Armed Forces are planning to deploy their latest American weapons systems
including M777 ultra-light howitzers and Chinook heavy-lift helicopters in the war games
to be carried out in Arunachal Pradesh, close to borders with China.

Codenamed HimVijay, this exercise planned by India in the northeast will mainly test the
war fighting abilities of the newly-raised 17 Mountain Strike Corps in Arunachal
Pradesh. The exercise will also involve the Indian Air Force (IAF) which would provide
the aerial elements for the real war-type drills.

Samudra Laksamana

The Indian Navy and Malaysian Navy had recently participated in the bilateral exercise
‘Samudra Laksamana’.

Pulikali

Also known as Tiger Dance, it is a folk art of Kerala, performed on the fourth day of
Onam festival.

Artists wear a tiger mask, paint their bodies like tigers and dance to the rhythm of
traditional percussion instruments such as thakil, udukku and chenda.

© IAS.NETWORK
38
Vulture Conservation and Breeding Centres

● The VCBC were started in 2004 when the vulture population had already crashed
significantly, almost by 99 %.
● At present there are nine (VCBC) in India, of which three are directly
administered by Bombay Natural History Society (BNHS).
● The three species of vultures bred in the VCBC are the White-backed,
Long-billed and the Slender-billed vulture.
● The objective of the VCBCs was not only to look after the vultures and breed
them in captivity, but also to release them into the wild.
● The first objective of the VCBC was to produce a few hundred pairs of each of
the three species of the endangered vultures.

Houthis

Is a rebel Shia group of Yemen that is backed by Iran, bombed the Abqaiq plant as well
as the Khurais oil field in Saudi Arabia using drones.

Nirvik Scheme
● To enhance the loan availability of exporters, and the MSME sector the Export
Guarantee Corporation of India (ECGC) has launched a new scheme called
‘Nirvik’.
● To revive the export sector, Commerce Ministry also launched the common
digital platform for the issuance of certificates of origin

Nirvik Scheme

● If there is any loss, then ECGC provided credit guarantee of up to 60% loss
approximately.
● Now under new scheme Nirvik consumers and exporters will covered up to 90%
and if there is any loss then in that case ECGC will refund 90% to the banks
including principal and interest.

© IAS.NETWORK
39
● Both pre and post shipment credit will also be covered under the new scheme.
● Banks will get up to 50 % within 30 days of complain lodge.
● Enhanced cover will ensure that Foreign and Rupee export credit interest rates
will be below 4% and 8% respectively for exporters.
● The scheme envisages simplified procedure for settlement of claim and for
provisional payment up to 50% within 30 days on production of proof of end-use
of the advances in default by the Insured Bank.

Electronic Certificates of Origin (CoO)

● This platform will be a single access point for all exporters, for all Free Trade
Agreements (FTAs)/ Preferential Trade Agreements (PTAs) and for all agencies
concerned.
● As we know, for exports to countries with which India has free trade agreements
(FTA), exporters have to show a certificate that the consignment originated in
India.
● With the launch of this platform, these certificates can be obtained online and all
the issuing authorities will be on the same portal.
● Certificate of Origin will be issued electronically which can be in paperless format
if agreed to by the partner countries.
● Authorities of partner countries will be able to verify the authenticity of certificates
from the website.

Report on ‘​hidden’ environmental, health and poverty costs

Food and Land use Coalition (FOLU)​, a collaboration of food, farming and green
research groups, has released a report on ‘hidden’ environmental, health and poverty
costs.

​ unar Reconnaissance Orbiter (LRO)


L

© IAS.NETWORK
40
ISRO is attempting to figure out what happened to Chandrayaan-2’s Vikram with the
help of NASA’s Lunar Reconnaissance Orbiter (LRO).

Lunar Reconnaissance Orbiter (LRO)

It is a NASA mission to the moon within the Lunar Precursor and Robotic Program
(LPRP) in preparation for future manned missions to the moon and beyond (Mars).

LRO is the first mission of NASA’s `New Vision for Space Exploration’.

The objectives of LRO are to:

● Identify potential lunar resources.


● Gather detailed maps of the lunar surface.
● Collect data on the moon’s radiation levels.
● Study the moons polar regions for resources that could be used in future manned
missions or robotic sample return missions.
● Provide measurements to characterize future robotic explorers, human lunar
landing sites and to derive measurements that can be used directly in support of
future Lunar Human Exploration Systems.

Survey of India

Survey of India (SoI) will for the first time rely on drones to map the country.

About SoI:

● It is the National Survey and Mapping Organization of the country under the
Department of Science & Technology.
● It is the oldest scientific department of the govt. of India set up in 1767.

Paraquat herbicide

© IAS.NETWORK
41
Paraquat

● Paraquat is a toxic chemical that is widely used as an herbicide (plant killer),


primarily for weed and grass control.
● It has been banned in 32 countries including Switzerland, where herbicide
producing company Sygenta is based.
● Paraquat also figures on the list of 99 pesticides and herbicides the Supreme
Court to ban in an ongoing case.
● Paraquat dichloride is being used for 25 crops in India, whereas it is approved to
be used on only nine crops by the Central Insecticide Board and Registration
Committee. This is a violation of the Indian Insecticides Act.
● So far in India, only Kerala has banned the herbicide.
● Another violation: since farmers can’t and don’t read the label on paraquat
containers, retailers sell paraquat in plastic carry bags and refill bottles.

The Ministry of Home Affairs has notified new rules for those receiving
foreign funding.

As per the new rules:

1. Every member of an NGO must now, under oath, through an affidavit, certify that
they have never been involved in “diverting” foreign funds or propagating
“sedition” or “advocating violent means”.
2. Earlier, the applicant or director-level person was to given this declaration but
now all members of the NGO need to be given this declaration.
Earlier, as per the market value of the gift item in India was Rs. 25,000, now it
has been raised to Rs. one lakh.
3. It is mandatory for the office bearers and key functionaries and members of the
NGOs to certify that they have not been “prosecuted or convicted” for
“conversion” from one faith to another and for creating “communal tension and
disharmony”.

​Regulation of Foreign Funding:

© IAS.NETWORK
42
The Foreign Contribution (Regulation) Act, 2010 and rules framed under it (the “FCRA”
or “Act”) regulate the receipt and usage of foreign contribution by non-governmental
organisations (“NGOs”) in India. Since the Act is internal security legislation, despite
being a law related to financial legislation, it falls into the purview of Home Ministry and
not the Reserve Bank of India (RBI).

International Migrant Stock 2019

Population Division of the UN Department of Economic and Social Affairs has released
the International Migrant Stock 2019.

UN prepared this report with the help of population censuses, population registers and
nationally representative surveys.

Report

1. India is the leading country of origin of international migrants in 2019 with a 17.5
million-strong diaspora.
2. Most of the international migrants came from Bangladesh, Pakistan and Nepal.
3. The number of migrants globally reached an estimated 272 million.
4. After India, Migrants from Mexico makes the second largest diaspora (11.8
million), followed by China (10.7 million), Russia (10.5 million), Syria (8.2 million),
Bangladesh (7.8 million), Pakistan (6.3 million), Ukraine (5.9 million), the
Philippines (5.4 million) and Afghanistan (5.1 million).
5. In Europe, 82 million migrants were settled there, followed by North America (59
million) and North Africa and Western Asia (49 million).
6. USA is hosting the largest number of international migrants (51 million) while
Saudi Arabia (13 million), Russia (12 million), England (10 million), France (8
million) and Italy (6 million) are also holding the large numbers of migrants.

Women migrants:

© IAS.NETWORK
43
1. The share of women and girls migrants fell slightly from 49% to 48% in the year
2019. Northern America had the highest (52%) number of women migrants.
However, Europe (51%), Sub-Saharan Africa (47 %) and Northern Africa and
Western Asia (36 %) also experienced the issue of women migrants.

Rustom- 2

● Rustom 2 drone is a medium-altitude, long-endurance unmanned aerial vehicle


developed by DRDO.
● The objective of this drone is to carry out surveillance for the armed forces with
an endurance of 24 hours.
● The drone was developed for use by all three services of the Indian armed
forces, primarily for intelligence, surveillance and reconnaissance (ISR)
operations.

International Migrant Stock 2019

The data is released by the UN DESA’s Population Division.


● Data shows that the number of international migrants in the world had reached
an estimated 272 million 2019 — 51 million more than in 2010.
● The percentage of international migrants of the total global population has
increased to 3.5% from 2.8% in 2000.

India at the top

● While India remained as the top source of international migrants, the number of
migrants living in India saw a slight decline from 5.24 million in 2015 to an
estimated 5.15 million in 2019 – both 0.4% of the total population of the country.
● Bangladesh was the leading country of origin for migrants in India.

© IAS.NETWORK
44
Coal gasification

Coal gasification is the process of producing syngas–a mixture consisting primarily of


carbon monoxide (CO), hydrogen (H​2​), carbon dioxide (CO​2​), natural gas (CH​4​) , and
water vapour (H​2​O)–from ​coal​ and water, air and/or oxygen. Historically, ​coal​ was
gasified to produce ​coal​ gas, also known as "town gas".

What is underground coal gasification process?


• Underground coal gasification (UCG) is an industrial process which converts coal into
product gas (predominant product gases are methane, hydrogen, carbon monoxide and
carbon dioxide)
• UCG is an in-situ gasification process carried out in non-mined coal seams using
injection of oxidants​,​ and bringing the product gas to surface through production wells
drilled from the surface.

New FCRA rules


● In a notification, the ministry announced the changes in the Foreign Contribution
(Regulation) Rules, 2011, which include that individuals need not declare
personal gifts to the tune of ₹1 lakh anymore.
● Earlier, gifts worth more than ₹25,000 were required to be declared.
● The office bearers, key functionaries and members of the organizations will have
to declare that they are neither prosecuted nor convicted of religious conversion
nor charged with sedition.
● It is also mandatory for office bearers and key functionaries and members to
certify that they have not been “prosecuted or convicted” for “conversion” from
one faith to another and for creating “communal tension and disharmony”.
● Earlier, as per the FCRA 2010, only applicants such as directors who sought
permission to receive foreign funds were required to make such a declaration.
● Now, every member of an NGO must, under oath, through an affidavit, declare
that they have never been involved in diverting foreign funds or “sedition” or
“advocating violent means”.

© IAS.NETWORK
45
About FCRA

● Government of India enacted the Foreign Contribution (Regulation) Act (FCRA)


in the year 1976 with an objective of regulating the acceptance and utilization of
foreign contribution.
● The act was majorly modified in 2010 with several amendments because many
NGOs were found using illegal use of foreign funding.

Access to internet is a basic right, says Kerala High Court


The Kerala High Court held that the right to have access to the Internet is part of the
fundamental right to education as well as the right to privacy under Article 21 of the
Constitution.

Atlantic Meridional Overturning Circulation (AMOC)

● AMOC is sometimes referred to as the “Atlantic conveyor belt”.

© IAS.NETWORK
46
● It is one of the Earth’s largest water circulation systems where ocean currents
move warm, salty water from the tropics to regions further north, such as western
Europe and sends colder water south.
● It aids in distributing heat and energy around the earth, as the warm water it
carries releases heat into the atmosphere, and in absorbing and storing
atmospheric carbon.
● The AMOC is a large system of ocean currents, like a conveyor belt, driven by
differences in temperature and salt content – the water’s density.
● As warm water flows northwards it cools and some evaporation occurs, which
increases the amount of salt. Low temperature and a high salt content make the
water denser, and this dense water sinks deep into the ocean.
● The cold, dense water slowly spreads southwards, several kilometres below the
surface. Eventually, it gets pulled back to the surface and warms in a process
called “upwelling” and the circulation is complete.

SITMEX - 19

Sea Phase of maiden ​SITMEX - 19 (Singapore India Thailand Maritime Exercise)​,


between Indian Navy, Republic of Singapore Navy (RSN) and Royal Thai Navy (RTN)
recently commenced in the Andaman Sea.

Digital Payment Abhiyan

Nasscom’s ​Data Security Council of India (DSCI) ​has collaborated with ​Union
Ministry of Electronics and IT (MeitY) and Google India​ to launch nationwide
awareness campaign ‘​Digital Payment Abhiyan​’.

● It aims at increasing awareness about cashless payment, educate end-users on


the benefits of making digital payments, online financial security and urge them
to adopt security and safety best practices.
● It is pan-India campaign crafted in seven languages — Hindi, English, Tamil,
Telugu, Kannada, Bengali and Marathi.
● It will engage with users and make them aware of the dos and don’ts for different
payment channels including UPI, wallets, cards as well as netbanking and mobile
banking

© IAS.NETWORK
47
NEAT Scheme

● Ministry of HRD has announced a new PPP Scheme, National Educational


Alliance for Technology (NEAT) for using technology for better learning outcomes
in Higher Education
● The objective is to use Artificial Intelligence to make learning more personalized
and customised as per the requirements of the learner.
● This requires development of technologies in Adaptive Learning to address the
diversity of learners.
● There are a number of start-up companies developing this and MHRD would like
to recognise such efforts and bring them under a common platform so that
learners can access it easily.
● Educating the youth is a National effort and MHRD proposes to create a National
Alliance with such technology developing EdTech Companies through a PPP
model.

Implementation

● AICTE would be the implementing agency for NEAT programme.


● The scheme shall be administered under the guidance of an Apex Committee
constituted by MHRD.
● Independent Expert Committees would be constituted for evaluating and
selecting the EdTech solutions.

Aditya- L1 mission

It is India’s first solar mission.It will study the sun’s outer most layers, the corona and
the chromospheres and collect data about coronal mass ejection, which will also yield
information for space weather prediction.​The Indian Space Research Organization is
planning to launch Aditya- L1 mission .

© IAS.NETWORK
48
Significance of the mission​: ​The data from Aditya mission will be immensely helpful in
discriminating between different models for the origin of solar storms and also for
constraining how the storms evolve and what path they take through the interplanetary
space from the Sun to the Earth.

Position of the satellite​: In order to get the best science from the sun, continuous
viewing of the sun is preferred without any occultation/ eclipses and hence, Aditya- L1
satellite will be placed in the halo orbit around the Lagrangian point 1 (L1) of the
sun-earth system.

Gandan Tegchenling Monastery

Location-​ Ulaanbaatar.

● Gandan Tegchenling Monastery is a prominent centre of Mongolian Buddhists


and a treasure house of valuable Buddhist heritage.
● It hosted the 11th General Assembly of Asian Buddhist Conference for Peace
(ABCP) from 21-23 June 2019 marking the 50th anniversary of the Conference

Fridays for Future movement

The school strike for climate also known variously as Fridays for Future (FFF), Youth​ ​for
Climate and Youth Strike 4 Climate is an international movement of school students
who take time off from class to participate in ​demonstrations​ to demand ​action​ to
prevent further ​global warming​ and climate chang​e.

Emergency Response Support System

● ERSS is one of the key projects of the Union MHA under Nirbhaya Fund.
● It has been designed to play a pivotal role in mitigation or preventing escalation
of crime, especially against women and children.

© IAS.NETWORK
49
● ERSS provides a single emergency number (112), computer aided dispatch of
field resources to the location of distress.
● Citizens can send their emergency information through call, sms, email and
through the 112 India mobile app.
● The ‘Dial 112’ emergency response service is an initiative to strengthen proactive
community policing that would end confusion amongst distress callers, who at
times end up dialling 100 in fire or medical emergency cases.

Waste Management Accelerator for Aspiring Women Entrepreneurs


(WAWE Summit 2019)

Launched by HRD ministry

About WAWE Summit 2019

1. The WAWE (Waste Management Accelerators for Aspire Women Entrepreneurs)


summit is scheduled to be held in November-December 2019.
2. It will be jointly organised by All India Council for Technical Education (AICTE)
and Indian Institute of Waste Management (IIWM) at Jaipur, Rajasthan. IIWM
and AICTE will be registering interested participation and guiding them to
connect from ‘Start Up India to Stand Up India’.
3. Theme: Make your own bag– empowering women to take up income generation
activity and entrepreneurship in waste management by making a business out of
this record creating concept.
4. This conclave will be the largest gathering of young women students to promote
entrepreneurship in waste management and providing alternatives to single use
plastic carry bags. Such initiatives are important in order to raise technical
education in country to world standards.

Beyond Visual Range Air-to-Air Missile (BVRAAM) ‘Astra’

© IAS.NETWORK
50
1. It is a state-of-the-art BVRAAM.
2. It has range of more than 100 kms.
3. It also has modern guidance and navigation techniques.
4. The missile has midcourse guidance and Radio Frequency (RF) seeker based
terminal guidance to achieve target destruction with pin point accuracy.
5. It has been developed by DRDO, together with Indian Air Force (IAF). Hindustan
Aeronautics Limited (HAL) has played a major role in modifying aircraft for
weapon integration. More than 50 public and private industries have contributed
in building Astra weapon system.

Digital Payment Abhiyan: DSCI, MeitY and Google India join hands

Nasscom’s Data Security Council of India (DSCI) has collaborated with Union Ministry
of Electronics and IT (MeitY) and Google India to launch nationwide awareness
campaign ‘Digital Payment Abhiyan’.

About Digital Payment Abhiyan

1. It aims at increasing awareness about cashless payment, educate end-users on


the benefits of making digital payments, online financial security and urge them
to adopt security and safety best practices. It is pan-India campaign crafted in
seven languages — Hindi, English, Tamil, Telugu, Kannada, Bengali and
Marathi.
2. It will engage with users and make them aware of the dos and don’ts for different
payment channels including UPI, wallets, cards as well as netbanking and mobile
banking Implementation: To drive the campaign objectives and amplify outreach
to users across all states, NASSCOM’s DSCI has onboarded various digital
payments ecosystem partners. These partners include representation from
banking, card networks as well as fin-tech segment.

© IAS.NETWORK
51
AISHE

All India Survey on Higher Education (AISHE) report for 2018-19 was recently released
by the HRD ministry.

● The survey, undertaken as an annual, web-based, pan-India exercise on the


status of Higher Education since 2010-11, covers all the Higher Educational
Institutions in the country
● The survey collects data on several parameters like teachers, student enrolment,
programmes, examination results, education finance,​ infrastructure, etc.

Key findings:

1. Gender gap narrowing.


2. More Girls in two states: In Uttar Pradesh and Karnataka, there are now more
females in the age group of 18-23 enrolling for higher education than male
students.
3. Female enrolment improved from 47.6% in 2017-18 to 48.6% in 2018-19.
4. The gross enrolment ratio (GER) increased marginally – from 25.8 in 2017-18 to
26.3 in 2018-19.
5. In absolute terms, enrolment increased from 3.66 crore to 3.74 crore students in
the same period. GER for SCs has also shown a growth from 21.8 to 23.0 and
STs from 15.9 to 17.2.
Number of universities has grown from 903 in 2017-18 to 993 in 2018-19 and
total HEIs (higher educational institutions) from 49,964 to 51,649 in the same
period.
6. Number of faculty has also increased from 13.88 lakh to 14.16 lakh.
7. Preferred stream at PG level: While one-third of undergraduate students are
enrolled in humanities The highest number of students are enrolled in Arts
courses., management seems to be a preferred stream at the postgraduate (PG)
level. Science and engineering technology registered relatively more enrolment
in M. Phil and Ph.D programmes.

© IAS.NETWORK
52
8. Preferred stream at the UG level, 35.9% of the total enrolment was in arts/
humanities/social science, just 16.5% students are pursuing science, followed by
commerce with 14.1%. Engineering is the fourth choice.

PACEsetter Fund programme

PACEsetter fund:

● Constituted by India and the USA in 2015.


● It is a INR 50 crore (USD 7.9 million) fund jointly capitalized by the Governments
of the Republic of India and the United States of America.
● It is a joint fund to provide early-stage grant funding to accelerate the
commercialization of innovative off-grid clean energy products, systems, and
business models.
● The Fund will award grants for direct support of innovative technology, business
models, and programs including but not limited to: rural energy services
companies (full scale integrated operators); rural distribution
companies/franchisees; operations/maintenance companies; technology
implementers/ system integrators; and enterprises.

Head on Generation (HOG) technology

Railway Ministry is upgrading all existing ​Linke Hofmann Busch (LHB) ​coaches with the
Head on Generation (HOG) technology​.

Head on Generation (HOG) technology

● The system runs the train’s ‘hotel load’ (the load of air conditioning, lights, fans,
and pantry, etc.) by drawing electricity from the overhead electric lines through
the pantograph
● The HOG system is free of air and noise pollution.

© IAS.NETWORK
53
● The system would bring down yearly CO2 and NOx emissions, which according
to the press release are currently at 1724.6 tonnes/annum and 7.48
tonnes/annum respectively, to zero.

How is it different from the present EOG technology?

Under the End on Generation (EOG) system, the train’s ‘hotel load’ (the load of air
conditioning, lights, fans, and pantry, etc.) is provided with electricity from two large
diesel generator sets, which supply 3-phase power at 750 Volts 50 Hz to the entire
length of the train.

Each coach then picks up the power supply through a 60 KVA transformer, bringing
down the voltage to 110 volts at which level the equipment in the compartment is run.
The generator cars are attached to either end of the train, giving the system its name.

Benefits of HOG over EOG:

● Since the HOG-fitted trains do not require power from diesel generators at all,
they only have one emergency generator car attached, instead of two regular
generator cars.
● The extra space created would now be used for an LSLRD (LHB Second
Luggage, Guard & Divyaang Compartment)– meaning more passengers can be
accommodated.
● Cost savings would be significant: Once all LHB trains get the new system, NR
1390 crores would be saved every year.
● HOG system is free of air and noise pollution: It would bring down yearly CO2
and NOx emissions, which are currently at 1724.6 tonnes/annum and 7.48
tonnes/annum respectively, to zero.

Central Advisory Board of Education (CABE)

© IAS.NETWORK
54
CABE is the highest advisory body to advise the Central and State Governments in the
field of education.It consists of nominated members representing various interests in
addition to elected members from the Lok Sabha and the Rajya Sabha, and the
representatives of the Government of India, State Governments and UT
Administrations.

Pusa Yashasvi

● It is a new wheat variety unveiled recently by the Indian Agricultural Research


Institute (IARI).
● Also Called HD-3226, the new variety has a higher weighted average yield
compare to predecessors.
● It also has higher content of protein and gluten (which contributes to strength and
elasticity of the dough), apart from more zinc (at 36.8 parts per million.
● It has high levels of resistance against all major rust fungi — yellow/stripe,
brown/leaf and black/stem.

Sagittarius A*

● It is a supermassive black hole that sits 26,000 light years away from Earth, near
the Galactic Centre, or the centre of the Milky Way.
● It is one of the few black holes where we can witness the flow of matter nearby.
● Since the discovery of Sagittarius A* 24 years ago, it has been fairly calm.

Ganga Data Collector

● Dehradun based Wildlife Institute of India (WII) has launched a mobile


application “Ganga Data Collector”.
● This is under the “Biodiversity and Ganga Conservation” project initiated by the
National Mission for Clean Ganga (NMCG) of Union Ministry of Jal Shakti.
● The app will provide field researchers with a complete data entry solution to
monitor the aquatic population in the river.
● The scientists of WII, Ganga Prahri volunteers and staff of the forest department
in the Ganga Basin will use this application on their mobile phones.

© IAS.NETWORK
55
● The Ganga basin covers 11 states including Uttarakhand, Uttar Pradesh,
Madhya Pradesh, Rajasthan, Haryana, Himachal Pradesh, Chhattisgarh,
Jharkhand, Bihar, West Bengal and Delhi.
● It will make faster collection of more authentic and accurate data related to water
quality and aquatic life in Ganga.

Shodh Shuddhi

● The union Ministry of HRD has launched the Plagiarism Detention Software
(PDS) “Shodh Shuddhi”.
● This service is being implemented by Information and Library Network
(INFLIBNET), an Inter University Centre (IUC) of UGC.
● PDS will significantly help to improve the quality of research outcome by ensuring
the originality of ideas and publication of the research scholars.

UMMID’ initiative

‘UMMID’ initiative to tackle inherited genetic diseases of new born babies.

UMMID (Unique Methods of Management and treatment of Inherited Disorders)


initiative:Department of Biotechnology has started the UMMID Initiative which is
designed on the concept of ‘Prevention is better than Cure’.

UMMID aims to create awareness about genetic disorders amongst clinicians and
establish molecular diagnostics in hospitals so that the fruits of developments in medical
genetics reach the patients in India.

Aims-

© IAS.NETWORK
56
1. Establish NIDAN (National Inherited Diseases Administration) Kendras to provide
counselling, prenatal testing and diagnosis, management, and multidisciplinary
care in Government Hospitals wherein the influx of patients is more.
2. Produce skilled clinicians in Human Genetics.
3. Undertake screening of pregnant women and new born babies for inherited
genetic diseases in hospitals at aspirational districts.

Participatory Guarantee Scheme (PGS)

● Union Agriculture Ministry’s PGS is a process of certifying organic products,


which ensures that their production takes place in accordance with laid-down
quality standards.
● The certification is in the form of a documented logo or a statement.
● According to the International Federation of Organic Agriculture Movements
(IFOAM), the Bonn-based global umbrella organisation for the organic agriculture
movement, PGSs are “locally focused quality assurance systems” that “certify
producers based on active participation of stakeholders and are built on a
foundation of trust, social networks and knowledge exchange”.

Emissio​ns Trading Scheme (ETS) of Gujarat


Gujarat government launched what is being described as the world’s first market for
trading in particulate matter emissions.

While trading mechanisms for pollution control do exist in many parts of the world, none
of them is for particulate matter emissions.

Emissions Trading Scheme (ETS)

● Launched in Surat, the Emissions Trading Scheme (ETS) is a regulatory tool that
is aimed at reducing the pollution load in an area and at the same time
minimising the cost of compliance for the industry.
● ETS is a market in which the traded commodity is particulate matter emissions.
● The Gujarat Pollution Control Board (GPCB) sets a cap on the total emission
load from all industries.

© IAS.NETWORK
57
● Various industries can buy and sell the ability to emit particulate matter, by
trading permits (in kilograms) under this cap.
● For this reason, ETS is also called a cap-and-trade market.

Trading process

● At the beginning of every one-month compliance period (during which one


emission permit is valid), 80 per cent of the total cap of 280 tonnes for that period
is distributed free to all participant units.
● These permits are allocated based on an industry’s emission sources (boilers,
heaters, generators) as this determines the amount of particulate matter emitted.
● GPCB will offer the remaining 20 per cent of the permits during the first auction of
the compliance period, at a floor price of Rs 5 per kilogram.
● Participating units may buy and sell permits among each other during the period.
● The price is not allowed to cross a ceiling of Rs 100 per kilogram or fall below Rs
5 per kg, both of which may be adjusted after a review.

Auctions

● These take place on the ETS-PM trading platform hosted by the National
Commodities and Derivatives Exchange e-Markets Limited (NeML).
● All participants must register a trading account with NeML. Transactions are
linked to the bank accounts of the users, who can view updates through these
accounts.
● There are two types of auctions. In the Uniform Price Auction, the week’s permit
price is discovered by participating members through bidding.
● Second, there is a continuous market between Wednesday where members will
buy and sell permits whose prices were fixed on Tuesday.
● For a true-up period of 2-7 days before the completion of the compliance period,
units may continue to buy and sell any remaining permits at the final auction
price to meet their compliance obligations.

Punitive actions for non-compliance

© IAS.NETWORK
58
● Based on permits held by units at the close of the compliance and true-up
periods, units will be declared compliant or non-compliant.
● Environmental damage compensation at Rs 200/kg will be imposed for emissions
in excess of a unit’s permit holdings at the end of the compliance period.
● This amount will be deducted from an environmental damage compensation
deposit that each unit has to submit before the start of the scheme — Rs 2 lakh
for small units, Rs 3 lakh for medium ones and Rs 10 lakh for large units.
● After any deduction, a unit will have to deposit extra money to meet that shortfall.
● To prevent any participant from hoarding permits, an upper limit has been set —
1.5 times the initial allocation for the compliance period, or 3 per cent of the
market cap for the compliance period.
● Also, no unit may sell more than 90 per cent of its initial allocation.

Clean Air Coalition and Clean Air Fund

● WHO is launching a “Clean Air Coalition” led by the Governments of Spain and
Peru, while a group of philanthropic organizations and foundations were poised
to launch a new “Clean Air Fund”.
● Both aim to spur investment in reducing sources of air pollution, which also
contribute to climate change.

Clean Air Coalition

● The Clean Air Coalition is being supported by the UN Secretary General’s Office
and the Climate and Clean Air Coalition of UN Environment.
● The fund brings together “a group of like-minded philanthropic foundations” which
have recognized that tackling air pollution will have “huge benefits for health as
well as for climate.”

Clean Air Fund

● The new Clean Air Fund aims to support projects that “democratize” air quality
data, making knowledge about air quality more widely accessible to large
numbers of people in cities.

© IAS.NETWORK
59
● The Fund works with a coalition of philanthropic foundation partners who have
interests in health, children, mobility, climate change, and equity, bringing them
together to strengthen their collective investment, voice and impact.

Methane-powered Rocket Engine


ISRO is developing two ‘LOx methane’ engines (liquid oxygen oxidiser and methane
fuel) engines.

Why use methane?

1. Methane, which can be synthesised with water and carbon dioxide in space, is
often described as the space fuel of the future.
2. Unsymmetrical Di-Methyl Hydrazine, along with Nitrogen tetroxide for oxidiser,
currently being used by ISRO, is said to be highly toxic and cancer-causing.
3. Whereas Methane, apart from being non-toxic, has a higher specific impulse
(which means one kg of the gas can life one kg of mass for a longer time), it is
easy to store, does not leave a residue upon burning, less bulky, and,
importantly, can be synthesised up in space

Climate Action Summit

The UN Climate Change Summit, 2019 was held in New York.The Climate Action
Summit aims to boost action to implement the Paris Agreement, which was signed in
2015.The first summit had taken place in California in 2018.

The Paris deal aims to strengthen the global response to the threat of climate change
by keeping a global temperature rise this century well below 2 degrees Celsius above
pre-industrial levels and to pursue efforts to limit the temperature increase even further
to 1.5 degrees Celsius.

Ladakhi Shondol dance

This dance has entered into the Guinness Book of World Records.

© IAS.NETWORK
60
It is known as the ‘royal dance of Ladakh’.It is famous dance which was earlier used to
be performed by artists for King of Ladakh on special occasion.

Tiger Triumph: India-US tri-services Exercise

This first-ever tri-Service exercise between India and US will focus on large-scale
amphibious Humanitarian Relief and Disaster Relief (HADR) operations. It will be held
along eastern coast in Bay of Bengal.

Global Internet Forum to Counter Terrorism (GIFCT)

● Formally established in July 2017 as a group of companies, dedicated to


disrupting terrorist abuse of members’ digital platforms.
● The original Forum was led by a rotating chair drawn from the founding four
companies—Facebook, Microsoft, Twitter and YouTube—and managed a
program of knowledge-sharing, technical collaboration and shared research.
● GIFCT as an independent organization was formally established in 2017​.

Christchurch Call to Action

● Launched during the “Online Extremism Summit” in Paris.


● Launched jointly by India, France, New Zealand, Canada and several other
countries.
● The initiative outlines collective and voluntary commitments from governments
and online service providers to address the issue of terrorist and violent extremist
content online.
● It calls for a free, open and secure internet to promote connectivity, enhance
social inclusiveness and foster economic growth.

​ he ‘Christchurch call to action’ initiative has been named after the New Zealand city
T
Christchurch, where over 50 people were killed and over 20 got injured in shootings at
two mosques.

© IAS.NETWORK
61
Leadership Group for Industry Transition

A ‘Leadership Group for Industry Transition’ was announced at the Global Climate
Action Summit to help guide the world’s heaviest greenhouse gas emitting industries
towards a low-carbon economy.

This new Leadership Group for Industry Transition is a global public-private effort to
ensure heavy industries and mobility companies can find a workable pathway to deliver
on the Paris Agreement.

“TB Harega Desh Jeetega” Campaign


● Union Ministry for Health and Family Welfare has recently launched a new ‘TB
Harega Desh Jeetega Campaign’.

About the campaign

● The campaign aims to improve and expand the reach of TB care services across
the country, by 2022.
● It has three pillars – clinical approach, public health component and active
community participation.
● There are some other supporting aspects of the campaign including – patient
support, private sector engagement, political and administrative commitment at
all levels.
● The government will ensure that all patients, at private or public hospitals,
receive free-of-cost and high-quality TB care.
● United Nations has marked 2030 as a global target to eliminate TB worldwide but
the GoI has made 2025 as its target to eliminate tuberculosis from the country.

Varaha

● It is an Indian Coast Guard Ship commissioned recently.


● It will enhance the surveillance and patrolling capabilities of Indian Coast Guard
and reinforce their role as ‘Sentinels of our Seas’.

© IAS.NETWORK
62
● It is the fourth in series of seven 98-m Offshore Patrol Vessels (OPV) of Indian
Coast Guard (ICG).
● It has been designed and built indigenously by Larsen & Toubro (L&T) at its
Katupalli ship building yard in North Chennai.

National Population Register project

● It is a Register of usual residents of the country.


● It is being prepared at the local (Village/sub-Town), sub-District, District, State
and National level under provisions of the Citizenship Act 1955 and the
Citizenship (Registration of Citizens and issue of National Identity Cards) Rules,
2003.
● It is mandatory for every usual resident of India to register in the NPR.
● Objectives: To create a comprehensive identity database of every usual resident
in the country.

Who is a usual resident

A usual resident is defined for the purposes of NPR as a person who has resided in a
local area for the past 6 months or more or a person who intends to reside in that area
for the next 6 months or more.

Components:

● The NPR database would contain demographic as well as biometric details.


● As per the provisions of the NPR, a resident identity card (RIC)will be issued to
individuals over the age of 18.
● This will be a chip-embedded smart card containing the demographic and
biometric attributes of each individual.
● The UID number will also be printed on the card.

How is NPR different from NRC?

● Unlike the NRC, the NPR is not a citizenship enumeration drive, as it would
record even a foreigner staying in a locality for more than six months.
● Only Assam will not be included, given the recently completed NRC.

© IAS.NETWORK
63
Project NETRA
● ISRO has initiated ‘Project NETRA’ – an early warning system in space to detect
debris and other hazards to Indian satellites.

Project NETRA (Network for space object Tracking and Analysis)

● The project will give India its own capability in space situational awareness (SSA)
like the other space powers — which is used to ‘predict’ threats from debris to
Indian satellites.
● NETRA’s eventual goal is to capture the GEO, or geostationary orbit, scene at
36,000 km where communication satellites operate.
● The space agency says our SSA will first be for low-earth orbits or LEO which
have remote-sensing spacecraft.
● Under NETRA the ISRO plans to put up many observational facilities: connected
radars, telescopes; data processing units and a control centre.
● They can, among others, spot, track and catalogue objects as small as 10 cm, up
to a range of 3,400 km and equal to a space orbit of around 2,000 km.
● The NETRA effort would make India a part of international efforts towards
tracking, warning about and mitigating space debris.

Forest-PLUS 2.0 Programme

● US Agency for International Development (USAID) and India’s MoEF&CC


officially launched Forest-PLUS 2.0 initiative.

Forest-PLUS 2.0

● It is a five-year programme initiated in December 2018 that focuses on


developing tools and techniques to bolster ecosystem management and
harnessing ecosystem services in forest landscape management.
● Forest-PLUS 2.0, the second set of pilot project is meant to enhance sustainable
forest landscape management after Forest-PLUS completed its five years in
2017.

© IAS.NETWORK
64
● The programme’s first set focused on capacity building to help India participate in
Reducing Emissions from Deforestation and forest Degradation (REDD+).
● It included four pilot projects in Sikkim, Rampur, Shivamogga and Hoshangabad.

About the programme

● Under these, field tests, innovative tools and approaches for Indian forest
management were developed.
● Promotion of bio-briquettes in Sikkim, introduction of solar heating systems in
Rampur and development of an agro-forestry model in Hoshangabad were some
of the achievements of this programme.
● Forest-PLUS 2.0 comprises pilot project in three landscapes — Gaya in Bihar,
Thiruvananthapuram in Kerala and Medak in Telangana.

Pacific Small Islands Developing States (PSIDS)


The PSIDS comprises of the 14 Pacific Island countries viz. The Cook Islands,
Federated States of Micronesia, Fiji, Kiribati, Nauru, Niue, Palau, Papua New Guinea,
Marshall Islands, Samoa, Solomon Islands, Tonga, Tuvalu, and Vanuatu.

About Small Island developing states (SIDS)

● SIDS is a group of small island countries that tend to share similar sustainable
development challenges.
● The challenges include small but growing populations, limited resources,
remoteness, susceptibility to natural disasters, vulnerability to external shocks,
excessive dependence on international trade, and fragile environments.
● Their growth and development is also held back by high communication, energy
and transportation costs and little to no opportunity to create economies of scale.
● These countries are across the globe in the Caribbean, the Pacific, Atlantic and
Indian Oceans, and the Mediterranean and South China Sea.

© IAS.NETWORK
65
PM – KISAN scheme

Update- ​T​he government has opened the Pradhan Mantri Kisan Samman Nidhi
(PM-Kisan) portal for self-registration by farmers.Allowing self-enrolment is expected to
help farmers in other States whose names have been missed out. Many States have
been slow in enrolling farmers for various reasons.

High-Temperature Proton Exchange Membrane (HTPEM) technology

India’s first indigenously developed high-temperature based Fuel Cell System was
recently introduced. It is a 5.0 kW fuel cell system that generates power in a green
manner.

● Developed under the Public-Private Partnership (PPP) model by the Council of


Scientific and Industrial Research (CSIR) in partnership with Indian industries.
● Built under India’s flagship programme named ‘New Millennium Indian
Technology Leadership Initiative (NMITLI)

Applications:

1. Suitable for distributed stationary power applications like; for small offices,
commercial units, data centers etc.; where highly reliable power is essential with
simultaneous requirement for air-conditioning.
2. It will also meet the requirement of efficient, clean and reliable backup power
generator for telecom towers, remote locations and strategic applications as well.
3. Replace Diesel Generating (DG) sets and help reduce India’s dependence on
crude oil.

What is HTPEM technology?

High Temperature Proton-Exchange-Membrane (HTPEM) is the core of the fuel cells


running above 150 °C. As in classical PEM fuel cells technology, Hydrogen is

© IAS.NETWORK
66
electrochemically split to proton and electron on anode. Proton is transported through
membrane to cathode while electricity is yielded in external circuit. At cathode protons
recombine with electron and reacts further with Oxygen to water and heat.

Special Report on the Ocean and Cryosphere in a Changing Climate

The Intergovernmental Panel on Climate Change (IPCC) released a special report-


‘Special Report on the Ocean and Cryosphere in a Changing Climate’.​ The report
underlines the dire changes taking place in oceans, glaciers and ice-deposits on land
and sea.

Key findings:

1. Over the 21st century, the ocean is projected to transition to unprecedented


conditions with increased temperatures, further ocean acidification, marine
heatwaves and more frequent extreme El Niño and La Niña events.
2. The global ocean has warmed unabated since 1970 and has taken up more than
90% of the excess heat in the climate system.
3. Since 1993, the rate of ocean warming has more than doubled.
4. Marine heatwaves have very likely doubled in frequency since 1982 and are
increasing in intensity.

Boiga thackerayi:

● It is a new species of snake that has been discovered in the Western Ghats in
Maharashtra.
● The species falls in the category commonly called cat snakes.
● It belongs to the genus Boiga and has Tiger like stripes on its body.
● This is the second species of Boiga after B. dightoni that is endemic to the
Western Ghats and the first new species of Boiga described after 125 years from
the Western Ghats.

IMD World Digital Competitiveness ranking 2019

© IAS.NETWORK
67
IMD World Digital Competitiveness Ranking measures the capacity and readiness of 63
economies to adopt and explore digital technologies as a key driver for economic
transformation in business, government and wider society.

● The ranking is produced by the IMD World Competitiveness Center.


● To evaluate an economy, WDCR examines three factors: Knowledge, the
capacity to understand and learn the new technologies; technology, the
competence to develop new digital innovations; and future readiness, the
preparedness for the coming developments.
● India rose from 48th place in 2018 to 44th rank this year.
● US was ranked as the world’s most digitally competitive economy, followed by
Singapore in the second place.

KAZIND-2019

It is a joint military exercise between India and Kazakhstan.The latest edition is


scheduled to be held in Pithoragarh district, Uttarkhand from 3-15 October 2019.

Focus: counter terrorism operation.

Govt launches new framework to sustain India’s ‘100% ODF status’


● The Union Jal Shakti Ministry’s Department of Drinking Water and Sanitation
(DDWS), launched a 10-year national rural sanitation strategy to sustain India’s
100 per cent Open Defecation Free (ODF).

About the framework

● The framework, to be in place from 2019 to 2029, will ensure that people sustain
their usage of toilets.
● It will also focus on proper implementation of solid and liquid waste management
(SLWM) — plastic waste, organic waste, grey water, and faecal sludge — in rural
areas.

© IAS.NETWORK
68
Steps to be undertaken

● They include the retrofitting of single pit toilets to twin pits or making provisions to
empty pits every five years, repair of defunct ones, and construction of soak pits
for septic tanks wherever not already present.
● A district-level training management unit (TMU) will be set up to provide oversight
and support to gram panchayats (GPs) so that they ensure the operation and
maintenance of sanitation infrastructure.
● The GPs are also supposed to conduct rapid assessment of water and sanitation
gaps.

Goldschmidtite
● A new, curious mineral has been discovered inside a diamond unearthed from a
mine in South Africa.

CARICOM Countries
● The Caribbean Community (CARICOM or CC) is an organisation of fifteen
Caribbean nations and dependencies. The organisation was established in 1973.
● They have primary objectives to promote economic integration and cooperation
among its members, to ensure that the benefits of integration are equitably
shared, and to coordinate foreign policy.

Infosys wins UN Global Climate Action Award in ‘Climate Neutral Now’


category

These awards are spearheaded by Momentum for Change initiative at UN Climate


Change. The projects which are awarded are recognized as an innovative solution that
addresses climate change as well as helps drive forward progress on many other
sustainable development goals, such as- innovation, gender equality and economic
opportunity. The UN Climate Change’s Momentum for Change initiative is implemented
with the support of The Rockefeller Foundation. It operates in partnership with World

© IAS.NETWORK
69
Economic Forum (WEF), donors supporting implementation of UN Climate Change’s
Gender Action Plan and Climate Neutral Now

MASCRADE 2019 organised by FICCI

MASCRADE 2019’- Movement Against Smuggling and Counterfeit Trade scheduled’,


an annual flagship FICCI (Federation of Indian Chambers of Commerce & Industry).

Climate Vulnerability Map of India

● The map is being developed under a joint project of the Department of Science
and Technology (DST) under the Union Ministry of Science and Technology and
Swiss Agency for Development and Cooperation (SDC).
● This research programme of DST is being implemented as part of the National
Mission for Sustaining the Himalayan Ecosystem (NMSHE) and National Mission
on Strategic Knowledge for Climate Change (NMSKCC).
● Such climate vulnerability atlas has already been developed for 12 states in the
Indian Himalayan Region, using a common framework.
● Now this methodology will be extended to non-Himalayan states so that we can
have a national level climate vulnerability profile for India.
● The atlas is expected to be ready by the middle of 2020.

Galo community in Arunachal Pradesh

© IAS.NETWORK
70
Members of the Galo community in Arunachal Pradesh can recall the name of
their ancestor from 20 generations, and this is made possible by their system of
naming.

INS Nilgiri

Navy’s first new stealth frigate, INS ‘Nilgiri’.

● It is the first ship of Project17A.


● Project 17A frigates is a design derivative of the Shivalik class stealth frigates
with much more advanced stealth features and indigenous weapons and
sensors.
● These frigates are being built using integrated construction methodology.
● The P17A frigates incorporate new design concepts for improved survivability,
sea keeping, stealth and ship manoeuvrability.

© IAS.NETWORK
71

You might also like